You are on page 1of 64

Elementary Linear Algebra

Elementary Linear Algebra

Kolman Hill
with Applications
Bernard Kolman David Hill
Ninth Edition

Ninth Edition
ISBN 978-1-29202-365-6

9 781292 023656
Elementary Linear Algebra
with Applications
Bernard Kolman David Hill
Ninth Edition
Pearson Education Limited
Edinburgh Gate
Harlow
Essex CM20 2JE
England and Associated Companies throughout the world

Visit us on the World Wide Web at: www.pearsoned.co.uk

© Pearson Education Limited 2014

All rights reserved. No part of this publication may be reproduced, stored in a retrieval system, or transmitted
in any form or by any means, electronic, mechanical, photocopying, recording or otherwise, without either the
prior written permission of the publisher or a licence permitting restricted copying in the United Kingdom
issued by the Copyright Licensing Agency Ltd, Saffron House, 6–10 Kirby Street, London EC1N 8TS.

All trademarks used herein are the property of their respective owners. The use of any trademark
in this text does not vest in the author or publisher any trademark ownership rights in such
trademarks, nor does the use of such trademarks imply any affiliation with or endorsement of this
book by such owners.

ISBN 10: 1-292-02365-1


ISBN 13: 978-1-292-02365-6

British Library Cataloguing-in-Publication Data


A catalogue record for this book is available from the British Library

Printed in the United States of America


P E A R S O N C U S T O M L I B R A R Y

Table of Contents

Glossary for Linear Algebra


Bernard Kolman/David R. Hill 1
Chapter 1. Linear Equations and Matrices
Bernard Kolman/David R. Hill 7
Chapter 2. Solving Linear Systems
Bernard Kolman/David R. Hill 92
Chapter 4. Real Vector Spaces
Bernard Kolman/David R. Hill 147
Chapter 5. Inner Product Spaces
Bernard Kolman/David R. Hill 260
Chapter 6. Linear Transformations and Matrices
Bernard Kolman/David R. Hill 333
Chapter 3. Determinants
Bernard Kolman/David R. Hill 407
Chapter 7. Eigenvalues and Eigenvectors
Bernard Kolman/David R. Hill 444
Chapter 8. Applications of Eigenvalues and Eigenvectors
Bernard Kolman/David R. Hill 489
Chapter 9. MATLAB for Linear Algebra
Bernard Kolman/David R. Hill 569
Chapter 10. MATLAB Exercises
Bernard Kolman/David R. Hill 605
Photo Credits
Bernard Kolman/David R. Hill 623
Index 625

I
II
GLOSSARY FOR LINEAR ALGEBRA

Additive inverse of a matrix: The additive inverse of an m ×n Closure properties: Let V be a given set, with members that
matrix A is an m × n matrix B such that A + B = O. Such a we call vectors, and two operations, one called vector addi-
matrix B is the negative of A, denoted −A, which is equal to tion, denoted ⊕, and the second called scalar multiplication,
(−1)A. denoted . We say that V is closed under ⊕, provided for u
 
Adjoint: For an n × n matrix A = ai j the adjoint of A, de- and v in V , u ⊕ v is a member of V . We say that V is closed
noted adj A is the transpose of the matrix formed under , provided for any real number k, k  u is a member
 by replacing
each entry by its cofactor Ai j ; that is, adj A = A ji . of V .
Angle between vectors: For nonzero vectors u and v in R n the Coefficient matrix: A linear system of m equations in n un-
angle θ between u and v is determined from the expression knowns has the form
u·v a11 x1 + a12 x2 + · · · + a1n xn = b1
cos(θ ) = .
uv a21 x1 + a22 x2 + · · · + a2n xn = b2
Augmented matrix: For the linear system Ax = b, the aug- .. .. .. ..
. . . .
mented matrix is formed by adjoining to the coefficient matrix
am1 x1 + am2 x2 + · · · + amn xn = bm .
 right side vector b. We expressed the augmented matrix
A the
as A b .
  The matrix
Back substitution: If U = u i j is an upper triangular matrix ⎡ ⎤
all of whose diagonal entries are not zero, then the linear sys- a11 a12 ··· a1n
tem U x = b can be solved by back substitution. The process ⎢ a21 a22 ··· a2n ⎥
⎢ ⎥
starts with the last equation and computes A=⎢ . .. .. ⎥
⎣ .. . . ⎦
bn am1 am2 ··· amn
xn = ;
u nn
is called the coefficient matrix of the linear system.
we use the next to last equation and compute  
Cofactor: For an n × n matrix A = ai j the cofactor Ai j
bn−1 − u n−1 n xn of ai j is defined as Ai j = (−1)i+ j det(Mi j ), where Mi j is the
xn−1 = ;
u n−1 n−1 i j-minor of A.
continuing in this fashion using the jth equation we compute Column rank: The column rank of a matrix A is the dimen-
sion of the column space of A or equivalently the number of

j+1
linearly independent columns of A.
bj − u jk xk
k=n Column space: The column space of a real m × n matrix A is
xj = . the subspace of R m spanned by the columns of A.
u jj
Complex vector space: A complex vector space V is a set,
Basis: A set of vectors S = {v1 , v2 , . . . , vk } from a vector with members that we call vectors, and two operations: one
space V is called a basis for V provided S spans V and S is a called vector addition, denoted ⊕, and the second called scalar
linearly independent set. multiplication, denoted . We require that V be closed under
Cauchy–Schwarz inequality: For vectors v and u in R n , the ⊕, that is, for u and v in V , u⊕v is a member of V ; in addition
Cauchy–Schwarz inequality says that the absolute value of the we require that V be closed under , that is, for any complex
dot product of v and u is less than or equal to the product of number k, k u is a member of V . There are 8 other properties
the lengths of v and u; that is, |v · u| ≤ vu. that must be satisfied before V with the two operations ⊕ and
Characteristic equation: For a square matrix A, its charac-  is called a complex vector space.
teristic equation is given by f (t) = det(A − t I ) = 0. Complex vector subspace: A subset W of a complex vector
Characteristic polynomial: For a square matrix A, its char- space V that is closed under addition and scalar multiplication
acteristic polynomial is given by f (t) = det(A − t I ). is called a complex subspace of V .

From Elementary Linear Algebra with Applications, Ninth Edition. Bernard Kolman, David R. Hill.
Copyright © 2008 by Pearson Education, Inc. All rights reserved.
1
A-32 Glossary For Linear Algebra

Components of a vector: The components of a vector v in R n Diagonalizable: A square matrix A is called diagonalizable
are its entries; ⎡ ⎤ provided it is similar to a diagonal matrix D; that is, there ex-
v1 ists a nonsingular matrix P such that P −1 A P = D.
⎢ v2 ⎥ Difference of matrices: The difference of the m × n matrices
⎢ ⎥
v = ⎢ . ⎥. A and B is denoted A − B and is equal to the sum A + (−1)B.
⎣ .. ⎦
The difference A − B is the m × n matrix whose entries are the
vn difference of corresponding entries of A and B.
Composite linear transformation: Let L 1 and L 2 be linear Difference of vectors: The difference of the vectors v and w
transformations with L 1 : V → W and L 2 : W → U . Then in a vector space V is denoted v − w, which is equal to the
the composition L 2 ◦ L 1 : V → U is a linear transformation sum v + (−1)w. If V = R n , then v − w is computed as the
and for v in V , we compute (L 2 ◦ L 1 )(v) = L 2 (L 1 (v)). difference of corresponding entries.
Computation of a determinant via reduction to triangular Dilation: The linear transformation L : R n → R n given by
form: For an n × n matrix A the determinant of A, denoted L(v) = kv, for k > 1, is called a dilation.
det(A) or |A|, can be computed with the aid of elementary row Dimension: The dimension of a nonzero vector space V is
operations as follows. Use elementary row operations on A, the number of vectors in a basis for V . The dimension of the
keeping track of the operations used, to obtain an upper trian- vector space {0} is defined as zero.
gular matrix. Using the changes in the determinant as the result
Distance between points (or vectors): The distance between
of applying a row operation and the fact that the determinant of
the points (u 1 , u 2 , . . . , u n ) and (v1 , v2 , . . . , vn ) is the length
an upper triangular matrix is the product of its diagonal entries,
of the vector u − v, where u = (u 1 , u 2 , . . . , u n ) and v =
we can obtain an appropriate expression for det(A).
(v1 , v2 , . . . , vn ), and is given by
Consistent linear system: A linear system Ax = b is called
consistent if the system has at least one solution. u − v = (u 1 − v1 )2 + (u 2 − v2 )2 + · · · + (u n − vn )2 .
Coordinates: The coordinates of a vector v in a vector space
V with ordered basis S = {v1 , v2 , . . . , vn } are the coefficients Thus we see that the distance between vectors in R n is also
c1 , c2 , . . . , cn such that v = c1 v1 +c2 v2 +· · ·+c u − v.
 n vn . We denote
the coordinates of v relative to the basis S by v S and write Dot product: For vectors v and w in R n the dot product of v
and w is also called the standard inner product or just the in-
⎡ ⎤ ner product of v and w. The dot product of v and w in R n is
c1
  ⎢ c2 ⎥ denoted v · w and is computed as
⎢ ⎥
v S = ⎢ . ⎥.
⎣ .. ⎦ v · w = v1 w1 + v2 w2 + · · · + vn wn .
cn Eigenspace: The set of all eigenvectors of a square matrix A
associated with a specified eigenvalue λ of A, together with
Cross product: The cross product of a pair of vectors u and v
the zero vector, is called the eigenspace associated with the
from R 3 is denoted u × v, and is computed as the determinant
eigenvalue λ.
i j k Eigenvalue: An eigenvalue of an n × n matrix A is a scalar λ
u1 u2 u3 , for which there exists a nonzero n-vector x such that Ax = λx.
v1 v2 v3 The vector x is an eigenvector associated with the eigenvalue
λ.
where i, j, and k are the unit vectors in the x-, y-, and z- Eigenvector: An eigenvector of an n ×n matrix A is a nonzero
directions, respectively. n-vector x such that Ax is a scalar multiple of x; that is, there
Defective matrix: A square matrix A is called defective if it exists some scalar λ such that Ax = λx. The scalar is an eigen-
has an eigenvalue of multiplicity m > 1 for which the associ- value of the matrix A.
ated eigenspace has a basis with fewer than m vectors. Elementary row operations: An elementary row operation on
Determinant: For an n × n matrix A the determinant of A, a matrix is any of the following three operations: (1) an inter-
denoted det(A) or |A|, is a scalar that is computed as the sum change of rows, (2) multiplying a row by a nonzero scalar, and
of all possible products of n entries of A each with its appro- (3) replacing a row by adding a scalar multiple of a different
priate sign, with exactly one entry from each row and exactly row to it.
one entry from each column. Equal matrices: The m × n matrices A and B are equal
 
Diagonal matrix: A square matrix A = ai j is called diago- provided corresponding entries are equal; that is, A = B if
nal provided ai j = 0 whenever i  = j. ai j = bi j , i = 1, 2, . . . , m, j = 1, 2, . . . , n.

2
Glossary For Linear Algebra A-33

Equal vectors: Vectors v and w in R n are equal provided cor- Identity matrix: The n × n identity matrix, denoted In , is a
responding entries are equal; that is, v = w if their correspond- diagonal matrix with diagonal entries of all 1s.
ing components are equal.
Inconsistent linear system: A linear system Ax = b that has
Finite-dimensional vector space: A vector space V that has a no solution is called inconsistent.
basis that is a finite subset of V is said to be finite dimensional.
  Infinite-dimensional vector space: A vector space V for
Forward substitution: If L = li j is a lower triangular ma-
which there is no finite subset of vectors that form a basis for
trix all of whose diagonal entries are not zero, then the linear
V is said to be infinite dimensional.
system Lx = b can be solved by forward substitution. The
process starts with the first equation and computes Inner product: For vectors v and w in R n the inner product of
v and w is also called the dot product or standard inner product
b1
x1 = ; of v and w. The inner product of v and w in R n is denoted v · w
l11 and is computed as
next we use the second equation and compute
b2 − l21 x1 v · w = v1 w1 + v2 w2 + · · · + vn wn .
x2 = ;
l22
Invariant subspace: A subspace W of a vector space V is said
continuing in this fashion using the jth equation we compute to be invariant under the linear transformation L : V → V ,

j−1 provided L(v) is in W for all vectors v in W .
bj − l jk xk Inverse linear transformation: See invertible linear transfor-
k=1
xj = . mation.
ljj
Inverse of a matrix: An n × n matrix A is said to have an
Fundamental vector spaces associated with a matrix: If A inverse provided there exists an n × n matrix B such that
is an m × n matrix there are four fundamental subspaces asso- AB = B A = I . We call B the inverse of A and denote it
ciated with A: (1) the null space of A, a subspace of R n ; (2) as A−1 . In this case, A is also called nonsingular.
the row space of A, a subspace of R n ; (3) the null space of A T ,
a subspace of R m ; and (4) the column space of A, a subspace Invertible linear transformation: A linear transformation
of R m . L : V → W is called invertible if there exists a linear transfor-
mation, denoted L −1 , such that L −1 (L(v)) = v, for all vectors
Gaussian elimination:  For the linear system Ax = b form the v in V and L(L −1 (w)) = w, for all vectors w in W .
augmented matrix A b . Compute the row echelon form
of the augmented matrix; then the solution can be computed Isometry: An isometry is a linear transformation L that
using back substitution. preserves the distance between pairs of vectors; that is,
L(v) − L(u) = v − u, for all vectors u and v. Since
Gauss–Jordan reduction:   the linear system Ax = b form
For
the augmented matrix A b . Compute the reduced row ech- an isometry preserves distances, it also preserves lengths; that
elon form of the augmented matrix; then the solution can be is, L(v) = v, for all vectors v.
computed using back substitution. Length (or magnitude) of a vector: The length of a vector v
General solution: The general solution of a consistent linear in R n is denoted v and is computed as the expression
system Ax = b is the set of all solutions to the system. If
b = 0, then the general solution is just the set of all solutions
to the homogeneous system Ax = 0, denoted xh . If b  = 0, v12 + v22 + · · · + vn2 .
then the general solution of the nonhomogeneous system con-
sists of a particular solution of Ax = b, denoted x p , together For a vector v in a vector space V on which an inner prod-
with xh ; that is, the general solution is expressed as x p + xh . uct (dot√product) is defined, the length of v is computed as
Gram–Schmidt process: The Gram–Schmidt process con- v = v · v.
verts a basis for a subspace into an orthonormal basis for the Linear combination: A linear combination of vectors
same subspace. v1 , v2 , . . . , vk from a vector space V is an expression of
Hermitian matrix: An n × n complex matrix A is called Her- the form c1 v1 + c2 v2 + · · · + ck vk , where the c1 , c2 , . . . , ck
T
mitian provided A = A. are scalars. A linear combination of the m × n matrices
A1 , A2 , . . . , Ak is given by c1 A1 + c2 A2 + · · · + ck Ak .
Homogeneous system: A homogeneous system is a linear sys-
tem in which the right side of each equation is zero. We denote Linear operator: A linear operator is a linear transformation
a homogeneous system by Ax = 0. L from a vector space to itself; that is, L : V → V .

3
A-34 Glossary For Linear Algebra

Linear system: A system of m linear equations in n unknowns Multiplicity of an eigenvalue: The multiplicity of an eigen-
x1 , x2 , . . . , xn is a set of linear equations in the n unknowns. value λ of a square matrix A is the number of times λ is a root
We express a linear system in matrix form as Ax = b, where of the characteristic polynomial of A.
A is the matrix of coefficients, x is the vector of unknowns, Natural (or standard) basis: The natural basis for R n is the
and b is the vector of right sides of the linear equations. (See set of vectors e j = column j (or, equivalently, row j) of the
coefficient matrix.) n × n identity matrix, j = 1, 2, . . . , n.
Linear transformation: A linear transformation L : V → W Negative of a vector: The negative of a vector u is a vector w
is a function assigning a unique vector L(v) in W to each vec- such that u + w = 0, the zero vector. We denote the negative
tor v in V such that two properties are satisfied: (1) L(u+v) = of u as −u = (−1)u.
L(u) + L(v), for every u and v in V , and (2) L(kv) = k L(v), Nonhomogeneous system: A linear system Ax = b is called
for every v in V and every scalar k. nonhomogeneous provided the vector b is not the zero vector.
Linearly dependent: A set of vectors S = {v1 , v2 , . . . , vn } is Nonsingular (or invertible) matrix: An n × n matrix A is
called linearly dependent provided there exists a linear combi- called nonsingular provided there exists an n × n matrix B
nation c1 v1 + c2 v2 + · · · + cn vn that produces the zero vector such that AB = B A = I . We call B the inverse of A and
when not all the coefficients are zero. denote it as A−1 .
Linearly independent: A set of vectors S = {v1 , v2 , . . . , vn } Nontrivial solution: A nontrivial solution of a linear system
is called linearly independent provided the only linear combi- Ax = b is any vector x containing at least one nonzero entry
nation c1 v1 + c2 v2 + · · · + cn vn that produces the zero vec- such that Ax = b.
tor is when all the coefficients are zero, that is, only when
c1 = c2 = · · · = cn = 0.  T  An n ×
Normal matrix:  nT complex
 matrix A is called normal
provided A A = A A .
Lower triangular matrix: A square matrix with zero entries
above its diagonal entries is called lower triangular. n-space: The set of all n-vectors is called n-space. For vectors
whose entries are real numbers we denote n-space as R n . For
LU-factorization (or LU-decomposition): An LU- a special case see 2-space.
factorization of a square matrix A expresses A as the product
of a lower triangular matrix L and an upper triangular matrix Nullity: The nullity of the matrix A is the dimension of the
U ; that is, A = LU . null space of A.
n-vector: A 1 × n or an n × 1 matrix is called an n-vector.
Main diagonal of a matrix: The main diagonal of an n × n
When n is understood, we refer to n-vectors merely as vectors.
matrix A is the set of entries a11 , a22 , . . . , ann .
One-to-one: A function f : S → T is said to be one-to-one
Matrix: An m ×n matrix A is a rectangular array of mn entries provided f (s1 )  = f (s2 ) whenever s1 and s2 are distinct el-
arranged in m rows and n columns. ements of S. A linear transformation L : V → W is called
 
Matrix addition: For m × n matrices A = ai j and B = one-to-one provided L is a one-to-one function.
 
bi j , the addition of A and B is performed by adding corre- Onto: A function f : S → T is said to be onto provided
   
sponding entries; that is, A + B = ai j + bi j . This is also for each member t of T there is some member s in S so that
called the sum of the matrices A and B. f (s) = t. A linear transformation L : V → W is called onto
Matrix representing a linear transformation: Let L : V → provided range L = W .
W be a linear transformation from an n-dimensional space V Ordered basis: A set of vectors S = {v1 , v2 , . . . , vk } in a vec-
to an m-dimensional space W . For a basis S = {v1 , v2 , . . . , vn } tor space V is called an ordered basis for V provided S is a
in V and a basis T = {w1 , w2 , . . . , wm } in W there basis for V and if we reorder the vectors in S, this new order-
 exists an
m × n matrix A, with column j of A = L(v j ) T such that ing of the vectors in S is considered a different basis for V .
the coordinates of L(x), for
 any x in V ,with  respect to the T Orthogonal basis: A basis for a vector space V that is also an
basis can be computed as L(x) T = A x S . We say A is the orthogonal set is called an orthogonal basis for V .
matrix representing the linear transformation L. Orthogonal complement: The orthogonal complement of a
Matrix transformation: For an m × n matrix A the function set S of vectors in a vector space V is the set of all vectors in
f defined by f (u) = Au for u in R n is called the matrix trans- V that are orthogonal to all vectors in S.
formation from R n to R m defined by the matrix A. Orthogonal matrix: A square matrix P is called orthogonal
 
Minor: Let A = ai j be an n × n matrix and Mi j the provided P −1 = P T .
(n − 1) × (n − 1) submatrix of A obtained by deleting the Orthogonal projection: For a vector v in a vector space V ,
ith row and jth column of A. The determinant det(Mi j ) is the orthogonal projection of v onto a subspace W of V with or-
called the minor of ai j . thonormal basis {w1 , w2 , . . . , wk } is the vector w in W , where

4
Glossary For Linear Algebra A-35

w = (v · w1 )w1 + (v · w2 )w2 + · · · + (v · wk )wk . Vector w is Rank: Since row rank A = column rank A, we just refer to
the vector in W that is closest to v. the rank of the matrix A as rank A. Equivalently, rank A =
Orthogonal set: A set of vectors S = {w1 , w2 , . . . , wk } from the number of linearly independent rows (columns) of A = the
a vector space V on which an inner product is defined is an number of leading 1s in the reduced row echelon form of A.
orthogonal set provided none of the vectors is the zero vector Real vector space: A real vector space V is a set, with mem-
and the inner product of any two different vectors is zero. bers that we call vectors and two operations: one is called vec-
Orthogonal vectors: A pair of vectors is called orthogonal tor addition, denoted ⊕, and the second called scalar multipli-
provided their dot (inner) product is zero. cation, denoted . We require that V be closed under ⊕; that
Orthogonally diagonalizable: A square matrix A is said to be is, for u and v in V , u ⊕ v is a member of V . In addition we
orthogonally diagonalizable provided there exists an orthogo- require that V be closed under ; that is, for any real number
nal matrix P such that P −1 A P is a diagonal matrix. That is, A k, k  u is a member of V . There are 8 other properties that
is similar to a diagonal matrix using an orthogonal matrix P. must be satisfied before V with the two operations ⊕ and  is
called a vector space.
Orthonormal basis: A basis for a vector space V that is also
an orthonormal set is called an orthonormal basis for V . Reduced row echelon form: A matrix is said to be in reduced
row echelon form provided it satisfies the following proper-
Orthonormal set: A set of vectors S = {w1 , w2 , . . . , wk }
ties: (1) All zero rows, if there are any, appear as bottom rows.
from a vector space V on which an inner product is defined
(2) The first nonzero entry in a nonzero row is a 1; it is called
is an orthonormal set provided each vector is a unit vector and
a leading 1. (3) For each nonzero row, the leading 1 appears
the inner product of any two different vectors is zero.
to the right and below any leading 1s in preceding rows. (4)
Parallel vectors: Two nonzero vectors are said to be parallel If a column contains a leading 1, then all other entries in that
if one is a scalar multiple of the other. column are zero.
Particular solution: A particular solution of a consistent lin- Reflection: The linear transformation L : R 2 → R 2 given by
ear system Ax = b is a vector x p containing no arbitrary con- L(x, y) = (x, −y) is called a reflection with respect to the x-
stants such that Ax p = b. axis. Similarly, L(x, y) = (−x, y) is called a reflection with
Partitioned matrix: A matrix that has been partitioned into respect to the y-axis.
submatrices by drawing horizontal lines between rows and/or Roots of the characteristic polynomial: For a square matrix
vertical lines between columns is called a partitioned matrix. A, the roots of its characteristic polynomial f (t) = det(A−t I )
There are many ways to partition a matrix. are the eigenvalues of A.
Perpendicular (or orthogonal) vectors: A pair of vectors is Rotation: The linear transformation L : R 2 → R 2 given by
said to be perpendicular or orthogonal provided their dot prod-
    
uct is zero. x cos(θ) − sin(θ) x
L =
Pivot: When using row operations on a matrix A, a pivot is a y sin(θ) cos(θ) y
nonzero entry of a row that is used to zero-out entries in the
column in which the pivot resides. is called a counterclockwise rotation in the plane by the
Positive definite: Matrix A is positive definite provided A is angle θ.
symmetric and all of its eigenvalues are positive. Row echelon form: A matrix is said to be in row echelon form
Powers of a matrix: For a square matrix A and nonnegative provided it satisfies the following properties: (1) All zero rows,
integer k, the kth power of A, denoted Ak , is the product of A if there are any, appear as bottom rows. (2) The first nonzero
with itself k times; Ak = A · A · · · · · A, where there are k entry in a nonzero row is a 1; it is called a leading 1. (3) For
factors. each nonzero row, the leading 1 appears to the right and below
any leading 1s in preceding rows.
Projection: The projection of a point P in a plane onto a
line L in the same plane is the point Q obtained by inter- Row equivalent: The m × n matrices A and B are row equiv-
secting the line L with the line through P that is perpendic- alent provided there exists a set of row operations that when
ular to L. The linear transformation L : R 3 → R 2 defined by performed on A yield B.
L(x, y, z) = (x, y) is called a projection of R 3 onto R 2 . (See Row rank: The row rank of matrix A is the dimension of the
also orthogonal projection.) row space of A or, equivalently, the number of linearly inde-
Range: The range of a function f : S → T is the set of pendent rows of A.
all members t of T such that there is a member s in S with Row space: The row space of a real m × n matrix A is the
f (s) = t. The range of a linear transformation L : V → W is subspace of R n spanned by the rows of A.
the set of all vectors in W that are images under L of vectors Scalar matrix: Matrix A is a scalar matrix provided A is a
in V . diagonal matrix with equal diagonal entries.

5
A-36 Glossary For Linear Algebra
 
Scalar multiple of a matrix: For an m × n matrix A = ai j Subspace: A subset W of a vector space V that is closed under
and scalar r , the scalar
 multiple of A by r gives the m × n addition and scalar multiplication is called a subspace of V .
matrix r A = rai j . Sum of vectors: The sum of two vectors is also called vec-
Scalar multiple of a vector: If v is in real vector space V , tor addition. In R n adding corresponding components of the
then for any real number k, a scalar, the scalar multiple of v by vectors performs the sum of two vectors. In a vector space V ,
k is denoted kv. If V = R n , then kv = (kv1 , kv2 , . . . , kvn ). u ⊕ v is computed using the definition of the operation ⊕.
Scalars: In a real vector space V the scalars are real numbers Summation notation: A compact notation to indicate the sum
and are used when we form scalar multiples kv, where v is of a set {a1 , a2 , . . . , an }; the sum of a1 through an is denoted
in V . Also, when we form linear combinations of vectors the  n
in summation notation as ai .
coefficients are scalars. i=1
Shear: A shear in the x-direction is defined by the matrix Symmetric matrix: A square real matrix A such that A = A T
transformation is called symmetric.
   Transition matrix: Let S = {v1 , v2 , . . . , vn } and T =
1 k u1
L(u) = , {w1 , w2 , . . . , wn } be bases for an n-dimensional vector space
0 1 u2
V . The transition matrix from the T -basis to the S-basis is
an n × n matrix, denoted PS←T , that converts the coordinates
where k is a scalar. Similarly, a shear in the y-direction is given
of a vector v relative to the   the coordinates of v
 T -basis into
by
   relative to the S-basis; v S = PS←T v T .
1 0 u1
L(u) = . Translation: Let T : V → V be defined by T (v) = v + b
k 1 u2
for all v in V and any fixed vector b in V . We call this the
Similar matrices: Matrices A and B are similar provided translation by the vector b.
there exists a nonsingular matrix P such that A = P −1 B P. Transpose of a matrix: The transpose of an m × n matrix A
Singular (or noninvertible) matrix: A matrix A that has no is the n × m matrix obtained by forming columns from each
inverse matrix is said to be singular. Any square matrix whose row of A. The transpose of A is denoted A T .
reduced row echelon form is not the identity matrix is singular. Trivial solution: The trivial solution of a homogeneous sys-
Skew symmetric matrix: A square real matrix A such that tem Ax = 0 is the zero vector.
A = −A T is called skew symmetric. 2-space: The set of all 2-vectors is called 2-space. For vectors
Solution space: The solution space of an m × n real homoge- whose entries are real numbers we denote 2-space as R 2 .
neous system Ax = 0 is the set W of all n-vectors x such that Unit vector: A vector of length 1 is called a unit vector.
A times x gives the zero vector. W is a subspace of R n . Unitary matrix: An n × n complex matrix A is called unitary
T
Solution to a homogeneous system: A solution to a homoge- provided A−1 = A .
neous system Ax = 0 is a vector x such that A times x gives Upper triangular matrix: A square matrix with zero entries
the zero vector. below its diagonal entries is called upper triangular.
Solution to a linear system: A solution to a linear system Vector: The generic name for any member of a vector space.
Ax = b is any vector x such that A times x gives the vector b. (See also 2-vector and n-vector.)
Span: The span of a set W = {w1 , w2 , . . . , wk }, denoted by Vector addition: The sum of two vectors is also called vec-
span W , from a vector space V is the set of all possible lin- tor addition. In R n adding corresponding components of the
ear combinations of the vectors w1 , w2 , . . . , wk . Span W is a vectors performs vector addition.
subspace of V. Zero matrix: A matrix with all zero entries is called the zero
Square matrix: A matrix with the same number of rows as matrix.
columns is called a square matrix. Zero polynomial: A polynomial all of whose coefficients are
Standard inner product: For vectors v and w in R n the stan- zero is called the zero polynomial.
dard inner product of v and w is also called the dot product of Zero subspace: The subspace consisting of exactly the zero
v and w, denoted v · w = v1 w1 + v2 w2 + · · · + vn wn . vector of a vector space is called the zero subspace.
Submatrix: A matrix obtained from a matrix A by deleting Zero vector: A vector with all zero entries is called the zero
rows and/or columns is called a submatrix of A. vector.

6
C H A P T E R

1 Linear Equations
and Matrices

1.1 Systems of Linear Equations


One of the most frequently recurring practical problems in many fields of study—
such as mathematics, physics, biology, chemistry, economics, all phases of engi-
neering, operations research, and the social sciences—is that of solving a system
of linear equations. The equation
a1 x1 + a2 x2 + · · · + an xn = b, (1)
which expresses the real or complex quantity b in terms of the unknowns x1 , x2 ,
. . . , xn and the real or complex constants a1 , a2 , . . . , an , is called a linear equa-
tion. In many applications we are given b and must find numbers x1 , x2 , . . . , xn
satisfying (1).
A solution to linear Equation (1) is a sequence of n numbers s1 , s2 , . . . , sn ,
which has the property that (1) is satisfied when x1 = s1 , x2 = s2 , . . . , xn = sn are
substituted in (1). Thus x1 = 2, x2 = 3, and x3 = −4 is a solution to the linear
equation
6x1 − 3x2 + 4x3 = −13,
because
6(2) − 3(3) + 4(−4) = −13.
More generally, a system of m linear equations in n unknowns, x1 , x2 , . . . , xn ,
or a linear system, is a set of m linear equations each in n unknowns. A linear

Note: Appendix A reviews some very basic material dealing with sets and functions. It can be
consulted at any time, as needed.

From Chapter 1 of Elementary Linear Algebra with Applications, Ninth Edition. Bernard Kolman,
David R. Hill. Copyright © 2008 by Pearson Education, Inc. All rights reserved.
7
2 Chapter 1 Linear Equations and Matrices

system can conveniently be written as

a11 x1 + a12 x2 + · · · + a1n xn = b1


a21 x1 + a22 x2 + · · · + a2n xn = b2
.. .. .. .. (2)
. . . .
am1 x1 + am2 x2 + · · · + amn xn = bm .
Thus the ith equation is
ai1 x1 + ai2 x2 + · · · + ain xn = bi .
In (2) the ai j are known constants. Given values of b1 , b2 , . . . , bm , we want to find
values of x1 , x2 , . . . , xn that will satisfy each equation in (2).
A solution to linear system (2) is a sequence of n numbers s1 , s2 , . . . , sn ,
which has the property that each equation in (2) is satisfied when x1 = s1 , x2 = s2 ,
. . . , xn = sn are substituted.
If the linear system (2) has no solution, it is said to be inconsistent; if it has
a solution, it is called consistent. If b1 = b2 = · · · = bm = 0, then (2) is
called a homogeneous system. Note that x1 = x2 = · · · = xn = 0 is always
a solution to a homogeneous system; it is called the trivial solution. A solution
to a homogeneous system in which not all of x1 , x2 , . . . , xn are zero is called a
nontrivial solution.
Consider another system of r linear equations in n unknowns:

c11 x1 + c12 x2 + · · · + c1n xn = d1


c21 x1 + c22 x2 + · · · + c2n xn = d2
.. .. .. .. (3)
. . . .
cr 1 x1 + cr 2 x2 + · · · + cr n xn = dr .
We say that (2) and (3) are equivalent if they both have exactly the same solutions.
The linear system
EXAMPLE 1
x1 − 3x2 = −7
(4)
2x1 + x2 = 7
has only the solution x1 = 2 and x2 = 3. The linear system

8x1 − 3x2 = 7
3x1 − 2x2 = 0 (5)
10x1 − 2x2 = 14
also has only the solution x1 = 2 and x2 = 3. Thus (4) and (5) are equivalent. ■
To find a solution to a linear system, we shall use a technique called the
method of elimination; that is, we eliminate some variables by adding a multiple
of one equation to another equation. Elimination merely amounts to the develop-
ment of a new linear system that is equivalent to the original system, but is much
simpler to solve. Readers have probably confined their earlier work in this area to

8
1.1 Systems of Linear Equations 3

linear systems in which m = n, that is, linear systems having as many equations
as unknowns. In this course we shall broaden our outlook by dealing with systems
in which we have m = n, m < n, and m > n. Indeed, there are numerous applica-
tions in which m  = n. If we deal with two, three, or four unknowns, we shall often
write them as x, y, z, and w. In this section we use the method of elimination as it
was studied in high school. In Section 2.2 we shall look at this method in a much
more systematic manner.
The director of a trust fund has $100,000 to invest. The rules of the trust state
EXAMPLE 2
that both a certificate of deposit (CD) and a long-term bond must be used. The
director’s goal is to have the trust yield $7800 on its investments for the year.
The CD chosen returns 5% per annum, and the bond 9%. The director determines
the amount x to invest in the CD and the amount y to invest in the bond as follows:
Since the total investment is $100,000, we must have x + y = 100,000. Since
the desired return is $7800, we obtain the equation 0.05x + 0.09y = 7800. Thus,
we have the linear system

x+ y = 100,000
(6)
0.05x + 0.09y = 7800.

To eliminate x, we add (−0.05) times the first equation to the second, obtaining

0.04y = 2800,

an equation having no x term. We have eliminated the unknown x. Then solving


for y, we have
y = 70,000,
and substituting into the first equation of (6), we obtain

x = 30,000.

To check that x = 30,000, y = 70,000 is a solution to (6), we verify that these


values of x and y satisfy each of the equations in the given linear system. This
solution is the only solution to (6); the system is consistent. The director of the
trust should invest $30,000 in the CD and $70,000 in the long-term bond. ■

Consider the linear system


EXAMPLE 3
x − 3y = −7
(7)
2x − 6y = 7.

Again, we decide to eliminate x. We add (−2) times the first equation to the
second one, obtaining
0 = 21,
which makes no sense. This means that (7) has no solution; it is inconsistent. We
could have come to the same conclusion from observing that in (7) the left side of
the second equation is twice the left side of the first equation, but the right side of
the second equation is not twice the right side of the first equation. ■

9
4 Chapter 1 Linear Equations and Matrices

Consider the linear system


EXAMPLE 4
x + 2y + 3z = 6
2x − 3y + 2z = 14 (8)
3x + y − z = −2.

To eliminate x, we add (−2) times the first equation to the second one and (−3)
times the first equation to the third one, obtaining

−7y − 4z = 2
(9)
−5y − 10z = −20.

This is a system of two equations in the unknowns y and z. We multiply the second
equation of (9) by − 15 , yielding

−7y − 4z = 2
y + 2z = 4,

which we write, by interchanging equations, as

y + 2z = 4
(10)
−7y − 4z = 2.

We now eliminate y in (10) by adding 7 times the first equation to the second one,
to obtain
10z = 30,

or

z = 3. (11)
Substituting this value of z into the first equation of (10), we find that y = −2.
Then substituting these values of y and z into the first equation of (8), we find that
x = 1. We observe further that our elimination procedure has actually produced
the linear system

x + 2y + 3z = 6
y + 2z = 4 (12)
z = 3,

obtained by using the first equations of (8) and (10) as well as (11). The importance
of this procedure is that, although the linear systems (8) and (12) are equivalent,
(12) has the advantage that it is easier to solve. ■

Consider the linear system


EXAMPLE 5
x + 2y − 3z = −4
(13)
2x + y − 3z = 4.

10
1.1 Systems of Linear Equations 5

Eliminating x, we add (−2) times the first equation to the second equation to get
−3y + 3z = 12. (14)
We must now solve (14). A solution is
y = z − 4,
where z can be any real number. Then from the first equation of (13),
x = −4 − 2y + 3z
= −4 − 2(z − 4) + 3z
= z + 4.
Thus a solution to the linear system (13) is
x = z+4
y = z−4
z = any real number.
This means that the linear system (13) has infinitely many solutions. Every time
we assign a value to z we obtain another solution to (13). Thus, if z = 1, then
x = 5, y = −3, and z=1
is a solution, while if z = −2, then
x = 2, y = −6, and z = −2
is another solution. ■

These examples suggest that a linear system may have a unique solution, no
solution, or infinitely many solutions.
Consider next a linear system of two equations in the unknowns x and y:

a1 x + a2 y = c1
(15)
b1 x + b2 y = c2 .
The graph of each of these equations is a straight line, which we denote by 1 and
2 , respectively. If x = s1 , y = s2 is a solution to the linear system (15), then the
point (s1 , s2 ) lies on both lines 1 and 2 . Conversely, if the point (s1 , s2 ) lies on
both lines 1 and 2 , then x = s1 , y = s2 is a solution to the linear system (15).
Thus we are led geometrically to the same three possibilities mentioned previously.
See Figure 1.1.
Next, consider a linear system of three equations in the unknowns x, y, and z:
a1 x + b1 y + c1 z = d1
a2 x + b2 y + c2 z = d2 (16)
a3 x + b3 y + c3 z = d3 .
The graph of each of these equations is a plane, denoted by P1 , P2 , and P3 , re-
spectively. As in the case of a linear system of two equations in two unknowns,

11
6 Chapter 1 Linear Equations and Matrices

(a) A unique solution. (b) No solution. (c) Infinitely many solutions.


y y y

2
2
x x x

1 2
FIGURE 1.1 1 1

the linear system in (16) can have infinitely many solutions, a unique solution, or
no solution. These situations are illustrated in Figure 1.2. For a more concrete
illustration of some of the possible cases, consider that two intersecting walls and
the ceiling (planes) of a room intersect in a unique point, a corner of the room,
so the linear system has a unique solution. Next, think of the planes as pages of
a book. Three pages of a book (held open) intersect in a straight line, the spine.
Thus, the linear system has infinitely many solutions. On the other hand, when the
book is closed, three pages of a book appear to be parallel and do not intersect, so
the linear system has no solution.

(a) A unique solution. (b) Infinitely many solutions. (c) No solution.

P1

P1
P3 P1 P3
P2
P2
P2 P3

FIGURE 1.2

If we examine the method of elimination more closely, we find that it involves


three manipulations that can be performed on a linear system to convert it into
an equivalent system. These manipulations are as follows:
1. Interchange the ith and jth equations.
2. Multiply an equation by a nonzero constant.
3. Replace the ith equation by c times the jth equation plus the ith equation,
i  = j. That is, replace
ai1 x1 + ai2 x2 + · · · + ain xn = bi
by
(ca j1 + ai1 )x1 + (ca j2 + ai2 )x2 + · · · + (ca jn + ain )xn = cb j + bi .

12
1.1 Systems of Linear Equations 7

It is not difficult to prove that performing these manipulations on a linear sys-


tem leads to an equivalent system. The next example proves this for the second
type of manipulation. Exercises 24 and 25 prove it for the first and third manipu-
lations, respectively.
Suppose that the ith equation of the linear system (2) is multiplied by the nonzero
EXAMPLE 6
constant c, producing the linear system
a11 x1 + a12 x2 + · · · + a1n xn = b1
a21 x1 + a22 x2 + · · · + a2n xn = b2
.. .. .. ..
. . . .
(17)
cai1 x1 + cai2 x2 + · · · + cain xn = cbi
.. .. .. ..
. . . .
am1 x1 + am2 x2 + · · · + amn xn = bm .
If x1 = s1 , x2 = s2 , . . . , xn = sn is a solution to (2), then it is a solution to all the
equations in (17), except possibly to the ith equation. For the ith equation we have
c(ai1 s1 + ai2 s2 + · · · + ain sn ) = cbi
or
cai1 s1 + cai2 s2 + · · · + cain sn = cbi .
Thus the ith equation of (17) is also satisfied. Hence every solution to (2) is also
a solution to (17). Conversely, every solution to (17) also satisfies (2). Hence (2)
and (17) are equivalent systems. ■
The following example gives an application leading to a linear system of two
equations in three unknowns:
(Production Planning) A manufacturer makes three different types of chemical
EXAMPLE 7
products: A, B, and C. Each product must go through two processing machines:
X and Y . The products require the following times in machines X and Y :
1. One ton of A requires 2 hours in machine X and 2 hours in machine Y .
2. One ton of B requires 3 hours in machine X and 2 hours in machine Y .
3. One ton of C requires 4 hours in machine X and 3 hours in machine Y .
Machine X is available 80 hours per week, and machine Y is available 60 hours
per week. Since management does not want to keep the expensive machines X and
Y idle, it would like to know how many tons of each product to make so that the
machines are fully utilized. It is assumed that the manufacturer can sell as much
of the products as is made.
To solve this problem, we let x1 , x2 , and x3 denote the number of tons of
products A, B, and C, respectively, to be made. The number of hours that machine
X will be used is
2x1 + 3x2 + 4x3 ,
which must equal 80. Thus we have
2x1 + 3x2 + 4x3 = 80.

13
8 Chapter 1 Linear Equations and Matrices

Similarly, the number of hours that machine Y will be used is 60, so we have
2x1 + 2x2 + 3x3 = 60.
Mathematically, our problem is to find nonnegative values of x1 , x2 , and x3 so that

2x1 + 3x2 + 4x3 = 80


2x1 + 2x2 + 3x3 = 60.
This linear system has infinitely many solutions. Following the method of
Example 4, we see that all solutions are given by
20 − x3
x1 =
2
x2 = 20 − x3
x3 = any real number such that 0 ≤ x3 ≤ 20,
since we must have x1 ≥ 0, x2 ≥ 0, and x3 ≥ 0. When x3 = 10, we have
x1 = 5, x2 = 10, x3 = 10
while
13
x1 = 2
, x2 = 13, x3 = 7
when x3 = 7. The reader should observe that one solution is just as good as the
other. There is no best solution unless additional information or restrictions are
given. ■

As you have probably already observed, the method of elimination has been
described, so far, in general terms. Thus we have not indicated any rules for select-
ing the unknowns to be eliminated. Before providing a very systematic descrip-
tion of the method of elimination, we introduce in the next section the notion of
a matrix. This will greatly simplify our notational problems and will enable us to
develop tools to solve many important applied problems.

Key Terms
Linear equation Consistent system Unique solution
Solution of a linear equation Homogeneous system No solution
Linear system Trivial solution Infinitely many solutions
Unknowns Nontrivial solution Manipulations on linear systems
Inconsistent system Equivalent systems Method of elimination

1.1 Exercises
In Exercises 1 through 14, solve each given linear system by 3. 3x + 2y + z = 2 4. x+ y= 5
the method of elimination. 4x + 2y + 2z = 8 3x + 3y = 10
x − y+ z=4
1. x + 2y = 8 2. 2x − 3y + 4z = −12 5. 2x + 4y + 6z = −12 6. x + y − 2z = 5
3x − 4y = 4 x − 2y + z = −5 2x − 3y − 4z = 15 2x + 3y + 4z = 2
3x + y + 2z = 1 3x + 4y + 5z = −8

14
1.1 Systems of Linear Equations 9

7. x + 4y − z = 12 8. 3x + 4y − z = 8 (a) Verify that x1 = 1, y1 = −1, z 1 = −1 is a solution.


3x + 8y − 2z = 4 6x + 8y − 2z = 3
(b) Verify that x2 = −2, y2 = 2, z 2 = 2 is a solution.
9. x + y + 3z = 12 10. x + y=1
2x + 2y + 6z = 6 2x − y = 5 (c) Is x = x1 + x2 = −1, y = y1 + y2 = 1, and
3x + 4y = 2 z = z 1 + z 2 = 1 a solution to the linear system?
11. 2x + 3y = 13 12. x − 5y = 6 (d) Is 3x, 3y, 3z, where x, y, and z are as in part (c), a
x − 2y = 3 3x + 2y = 1 solution to the linear system?
5x + 2y = 27 5x + 2y = 1 20. Without using the method of elimination, solve the linear
13. x + 3y = −4 14. 2x + 3y − z = 6 system
2x + 5y = −8 2x − y + 2z = −8 2x + y − 2z = −5
x + 3y = −5 3x − y + z = −7 3y + z = 7
15. Given the linear system z = 4.
2x − y = 5 21. Without using the method of elimination, solve the linear
4x − 2y = t, system
4x = 8
(a) Determine a particular value of t so that the system
is consistent. −2x + 3y = −1
(b) Determine a particular value of t so that the system 3x + 5y − 2z = 11.
is inconsistent. 22. Is there a value of r so that x = 1, y = 2, z = r is a
(c) How many different values of t can be selected in solution to the following linear system? If there is, find
part (b)? it.
16. Given the linear system 2x + 3y − z = 11
x − y + 2z = −7
3x + 4y = s 4x + y − 2z = 12.
6x + 8y = t,
23. Is there a value of r so that x = r , y = 2, z = 1 is a
(a) Determine particular values for s and t so that the
solution to the following linear system? If there is, find
system is consistent.
it.
(b) Determine particular values for s and t so that the 3x − 2z = 4
system is inconsistent.
x − 4y + z = −5
(c) What relationship between the values of s and t will
−2x + 3y + 2z = 9.
guarantee that the system is consistent?
17. Given the linear system 24. Show that the linear system obtained by interchanging
two equations in (2) is equivalent to (2).
x+ 2y = 10
3x + (6 + t)y = 30, 25. Show that the linear system obtained by adding a multi-
ple of an equation in (2) to another equation is equivalent
(a) Determine a particular value of t so that the system to (2).
has infinitely many solutions.
26. Describe the number of points that simultaneously lie in
(b) Determine a particular value of t so that the system
each of the three planes shown in each part of Figure 1.2.
has a unique solution.
(c) How many different values of t can be selected in 27. Describe the number of points that simultaneously lie in
part (b)? each of the three planes shown in each part of Figure 1.3.
18. Is every homogeneous linear system always consistent? 28. Let C1 and C2 be circles in the plane. Describe the num-
Explain. ber of possible points of intersection of C1 and C2 . Illus-
19. Given the linear system trate each case with a figure.
29. Let S1 and S2 be spheres in space. Describe the number
2x + 3y − z = 0
of possible points of intersection of S1 and S2 . Illustrate
x − 4y + 5z = 0, each case with a figure.

15
10 Chapter 1 Linear Equations and Matrices

P1 available 16 hours per day, how many tons of each type


P1
of developer can be produced so that the plants are fully
P2 P3
used?
P2
P3 34. Suppose that the three points (1, −5), (−1, 1), and (2, 7)
lie on the parabola p(x) = ax 2 + bx + c.
(a) (b) (a) Determine a linear system of three equations in three
unknowns that must be solved to find a, b, and c.
(b) Solve the linear system obtained in part (a) for a, b,
P1 and c.
P2 35. An inheritance of $24,000 is to be divided among three
trusts, with the second trust receiving twice as much as
the first trust. The three trusts pay interest annually at
the rates of 9%, 10%, and 6%, respectively, and return a
P3 total in interest of $2210 at the end of the first year. How
much was invested in each trust?
(c) 36. For the software you are using, determine the command
that “automatically” solves a linear system of equations.
FIGURE 1.3
37. Use the command from Exercise 36 to solve Exercises 3
30. An oil refinery produces low-sulfur and high-sulfur fuel. and 4, and compare the output with the results you ob-
Each ton of low-sulfur fuel requires 5 minutes in the tained by the method of elimination.
blending plant and 4 minutes in the refining plant; each
ton of high-sulfur fuel requires 4 minutes in the blending 38. Solve the linear system
plant and 2 minutes in the refining plant. If the blend- x + 12 y + 13 z = 1
ing plant is available for 3 hours and the refining plant is 1
available for 2 hours, how many tons of each type of fuel 2
x + 13 y + 14 z = 11
18
1
should be manufactured so that the plants are fully used? 3
x + 14 y + 15 z = 9
20
31. A plastics manufacturer makes two types of plastic: reg- by using your software. Compare the computed solution
ular and special. Each ton of regular plastic requires 2 with the exact solution x = 12 , y = 13 , z = 1.
hours in plant A and 5 hours in plant B; each ton of spe-
39. If your software includes access to a computer algebra
cial plastic requires 2 hours in plant A and 3 hours in
system (CAS), use it as follows:
plant B. If plant A is available 8 hours per day and plant
B is available 15 hours per day, how many tons of each (a) For the linear system in Exercise 38, replace the
type of plastic can be made daily so that the plants are fraction 12 with its decimal equivalent 0.5. Enter this
fully used? system into your software and use the appropriate
CAS commands to solve the system. Compare the
32. A dietician is preparing a meal consisting of foods A, B,
solution with that obtained in Exercise 38.
and C. Each ounce of food A contains 2 units of pro-
tein, 3 units of fat, and 4 units of carbohydrate. Each (b) In some CAS environments you can select the num-
ounce of food B contains 3 units of protein, 2 units of ber of digits to be used in the calculations. Perform
fat, and 1 unit of carbohydrate. Each ounce of food C part (a) with digit choices 2, 4, and 6 to see what
contains 3 units of protein, 3 units of fat, and 2 units of influence such selections have on the computed so-
carbohydrate. If the meal must provide exactly 25 units lution.
of protein, 24 units of fat, and 21 units of carbohydrate, 40. If your software includes access to a CAS and you can
how many ounces of each type of food should be used? select the number of digits used in calculations, do the
33. A manufacturer makes 2-minute, 6-minute, and 9-minute following: Enter the linear system
film developers. Each ton of 2-minute developer requires 0.71x + 0.21y = 0.92
6 minutes in plant A and 24 minutes in plant B. Each ton
of 6-minute developer requires 12 minutes in plant A and 0.23x + 0.58y = 0.81
12 minutes in plant B. Each ton of 9-minute developer re- into the program. Have the software solve the system
quires 12 minutes in plant A and 12 minutes in plant B. with digit choices 2, 5, 7, and 12. Briefly discuss any
If plant A is available 10 hours per day and plant B is variations in the solutions generated.

16
1.2 Matrices 11

1.2 Matrices
If we examine the method of elimination described in Section 1.1, we can make the
following observation: Only the numbers in front of the unknowns x1 , x2 , . . . , xn
and the numbers b1 , b2 , . . . , bm on the right side are being changed as we perform
the steps in the method of elimination. Thus we might think of looking for a way
of writing a linear system without having to carry along the unknowns. Matrices
enable us to do this—that is, to write linear systems in a compact form that makes
it easier to automate the elimination method by using computer software in order
to obtain a fast and efficient procedure for finding solutions. The use of matrices,
however, is not merely that of a convenient notation. We now develop operations
on matrices and will work with matrices according to the rules they obey; this will
enable us to solve systems of linear equations and to handle other computational
problems in a fast and efficient manner. Of course, as any good definition should
do, the notion of a matrix not only provides a new way of looking at old problems,
but also gives rise to a great many new questions, some of which we study in this
book.
An m ×n matrix A is a rectangular array of mn real or complex numbers arranged
DEFINITION 1.1
in m horizontal rows and n vertical columns:
⎡ ⎤
a11 a12 · · · · · · · · · a1n
⎢ a21 a22 · · · · · · · · · a2n ⎥
⎢ ⎥
⎢ .. .. .. ⎥
⎢ . . ··· ··· ··· . ⎥
⎢ ⎥
A=⎢ . .
.. ⎥
. . (1)
⎢ .. .. · · · · · · a ⎥  ith row
⎢ ij ⎥
⎢ .. .. .. ⎥
⎣ . . ··· ··· ··· . ⎦
am1 am2 · · · · · · · · · amn
6 jth column
The ith row of A is

ai1 ai2 · · · ain (1 ≤ i ≤ m);

the jth column of A is


⎡ ⎤
a1 j
⎢ a2 j ⎥
⎢ ⎥
⎢ .. ⎥ (1 ≤ j ≤ n).
⎣ . ⎦
am j

We shall say that A is m by n (written as m × n). If m = n, we say that A is a


square matrix of order n, and that the numbers a11 , a22 , . . . , ann form the main
diagonal of A. We refer to the number ai j , which is in the ith row and jth column
of A, as the i, jth element of A, or the (i, j) entry of A, and we often write (1) as

A = ai j .

17
12 Chapter 1 Linear Equations and Matrices

Let
EXAMPLE 1


1
1 2 3 1+i 4i
A= , B= , C = ⎣ −1 ⎦ ,
−1 0 1 2 − 3i −3
2
⎡ ⎤
1 1 0
D = ⎣2 0 1⎦ , E= 3 , F = −1 0 2 .
3 −1 2

Then A is a 2 × 3 matrix with a12 = 2, a13 = 3, a22 = 0, and a23 = 1; B is a 2 × 2


matrix with b11 = 1 + i, b12 = 4i, b21 = 2 − 3i, and b22 = −3; C is a 3 × 1 matrix
with c11 = 1, c21 = −1, and c31 = 2; D is a 3 × 3 matrix; E is a 1 × 1 matrix;
and F is a 1 × 3 matrix. In D, the elements d11 = 1, d22 = 0, and d33 = 2 form
the main diagonal. ■

For convenience, we focus much of our attention in the illustrative examples


and exercises in Chapters 1–6 on matrices and expressions containing only real
numbers. Complex numbers make a brief appearance in Chapter 7. An introduc-
tion to complex numbers, their properties, and examples and exercises showing
how complex numbers are used in linear algebra may be found in Appendix B.
An n×1 matrix is also called an n-vector and is denoted by lowercase boldface
letters. When n is understood, we refer to n-vectors merely as vectors. Vectors
are discussed at length in Section 4.1.
⎡ ⎤
1 ⎡ ⎤
⎢ 2⎥ 1
u=⎢ ⎥ ⎣ ⎦
⎣ −1 ⎦ is a 4-vector and v = −1 is a 3-vector. ■
EXAMPLE 2
3
0

The n-vector all of whose entries are zero is denoted by 0.


Observe that if A is an n × n matrix, then the rows of A are 1 × n matrices and
the columns of A are n × 1 matrices. The set of all n-vectors with real entries is
denoted by R n . Similarly, the set of all n-vectors with complex entries is denoted
by C n . As we have already pointed out, in the first six chapters of this book we
work almost entirely with vectors in R n .

(Tabular Display of Data) The following matrix gives the airline distances be-
EXAMPLE 3
tween the indicated cities (in statute miles):

London Madrid New York Tokyo


⎡ ⎤
London 0 785 3469 5959
Madrid ⎢ 785 0 3593 6706 ⎥
New York
⎣ 3469 3593 0 6757 ⎦
Tokyo 5959 6706 6757 0 ■

(Production) Suppose that a manufacturer has four plants, each of which makes
EXAMPLE 4
three products. If we let ai j denote the number of units of product i made by plant

18
1.2 Matrices 13

j in one week, then the 3 × 4 matrix

⎡ Plant 1 Plant 2 Plant 3 Plant 4 ⎤


Product 1 560 360 380 0
Product 2 ⎣ 340 450 420 80 ⎦
Product 3 280 270 210 380

gives the manufacturer’s production for the week. For example, plant 2 makes 270
units of product 3 in one week. ■

The windchill table that follows is a matrix.


EXAMPLE 5

F
15 10 5 0 −5 −10
mph
5 12 7 0 −5 −10 −15
10 −3 −9 −15 −22 −27 −34
15 −11 −18 −25 −31 −38 −45
20 −17 −24 −31 −39 −46 −53
A combination of air temperature and wind speed makes a body feel colder than
the actual temperature. For example, when the temperature is 10◦ F and the wind is
15 miles per hour, this causes a body heat loss equal to that when the temperature
is −18◦ F with no wind. ■

By a graph we mean a set of points called nodes or vertices, some of which are
EXAMPLE 6
connected by lines called edges. The nodes are usually labeled as P1 , P2 , . . . , Pk ,
and for now we allow an edge to be traveled in either direction. One mathematical
representation of a graph is constructed from a table. For example, the following
table represents the graph shown:

P1 P2 P3 P4 P3
P1 0 1 0 0
P2 1 0 1 1 P2
P3 0 1 0 1 P4
P4 0 1 1 0 P1

The (i, j) entry = 1 if there is an edge connecting vertex Pi to vertex P j ;


otherwise, the (i, j) entry = 0. The incidence matrix A is the k × k matrix
obtained by omitting the row and column labels from the preceding table. The
incidence matrix for the corresponding graph is
⎡ ⎤
0 1 0 0
⎢1 0 1 1⎥
A=⎢
⎣0
⎥.
1 0 1⎦
0 1 1 0 ■

19
14 Chapter 1 Linear Equations and Matrices

Internet search engines use matrices to keep track of the locations of informa-
tion, the type of information at a location, keywords that appear in the information,
and even the way websites link to one another. A large measure of the effective-
ness of the search engine Google c
is the manner in which matrices are used to
determine which sites are referenced by other sites. That is, instead of directly
keeping track of the information content of an actual web page or of an individual
search topic, Google’s matrix structure focuses on finding web pages that match
the search topic, and then presents a list of such pages in the order of their “impor-
tance.”
Suppose that there are n accessible web pages during a certain month. A
simple way to view a matrix that is part of Google’s scheme is to imagine an n × n
matrix A, called the “connectivity matrix,” that initially contains all zeros. To
build the connections, proceed as follows. When you detect that website j links
to website i, set entry ai j equal to one. Since n is quite large, in the billions, most
entries of the connectivity matrix A are zero. (Such a matrix is called sparse.)
If row i of A contains many ones, then there are many sites linking to site i.
Sites that are linked to by many other sites are considered more “important” (or to
have a higher rank) by the software driving the Google search engine. Such sites
would appear near the top of a list returned by a Google search on topics related
to the information on site i. Since Google updates its connectivity matrix about
every month, n increases over time and new links and sites are adjoined to the
connectivity matrix.
In Chapter 8 we elaborate a bit on the fundamental technique used for ranking
sites and give several examples related to the matrix concepts involved. Further
information can be found in the following sources:
1. Berry, Michael W., and Murray Browne. Understanding Search Engines—
Mathematical Modeling and Text Retrieval, 2d ed. Philadelphia: Siam, 2005.
2. www.google.com/technology/index.html
3. Moler, Cleve. “The World’s Largest Matrix Computation: Google’s PageRank
Is an Eigenvector of a Matrix of Order 2.7 Billion,” M ATLAB News and Notes,
October 2002, pp. 12–13.
Whenever a new object is introduced in mathematics, we must determine
when two such objects are equal. For example, in the set of all rational numbers,
the numbers 23 and 46 are called equal, although they have different representa-
tions. What we have in mind is the definition that a/b equals c/d when ad = bc.
Accordingly, we now have the following definition:
Two m × n matrices A = ai j and B = bi j are equal if they agree entry by
DEFINITION 1.2
entry, that is, if ai j = bi j for i = 1, 2, . . . , m and j = 1, 2, . . . , n.

The matrices
EXAMPLE 7
⎡ ⎤ ⎡ ⎤
1 2 −1 1 2 w
A = ⎣ 2 −3 4⎦ and B = ⎣2 x 4⎦
0 −4 5 y −4 z

are equal if and only if w = −1, x = −3, y = 0, and z = 5. ■

20
1.2 Matrices 15

■ Matrix Operations
We next define a number of operations that will produce new matrices out of given
matrices. When we are dealing with linear systems, for example, this will en-
able us to manipulate the matrices that arise and to avoid writing down systems
over and over again. These operations and manipulations are also useful in other
applications of matrices.

Matrix Addition
If A = ai j and B = bi j are both m×n matrices, then the sum A+B is an m×n
DEFINITION 1.3
matrix C = ci j defined by ci j = ai j + bi j , i = 1, 2, . . . , m; j = 1, 2, . . . , n.
Thus, to obtain the sum of A and B, we merely add corresponding entries.

Let
EXAMPLE 8
1 −2 3 0 2 1
A= and B= .
2 −1 4 1 3 −4
Then
1 + 0 −2 + 2 3+1 1 0 4
A+B = = .
2 + 1 −1 + 3 4 + (−4) 3 2 0 ■
(Production) A manufacturer of a certain product makes three models, A, B, and
EXAMPLE 9
C. Each model is partially made in factory F1 in Taiwan and then finished in factory
F2 in the United States. The total cost of each product consists of the manufactur-
ing cost and the shipping cost. Then the costs at each factory (in dollars) can be
described by the 3 × 2 matrices F1 and F2 :
Manufacturing Shipping
cost cost
⎡ ⎤
32 40 Model A
F1 = ⎣ 50 80 ⎦ Model B
70 20 Model C

Manufacturing Shipping
cost cost
⎡ ⎤
40 60 Model A
F2 = ⎣ 50 50 ⎦ Model B .
130 20 Model C

The matrix F1 + F2 gives the total manufacturing and shipping costs for each
product. Thus the total manufacturing and shipping costs of a model C product are
$200 and $40, respectively. ■

If x is an n-vector, then it is easy to show that x + 0 = x, where 0 is the


n-vector all of whose entries are zero. (See Exercise 16.)
It should be noted that the sum of the matrices A and B is defined only when
A and B have the same number of rows and the same number of columns, that is,
only when A and B are of the same size.

21
16 Chapter 1 Linear Equations and Matrices

We now make the convention that when A + B is written, both A and B are of
the same size.

The basic properties of matrix addition are considered in the next section and
are similar to those satisfied by the real numbers.

Scalar Multiplication
If A = ai j is an m × n matrix and r is a real number, then the scalar multiple of
DEFINITION 1.4
A by r , r A, is the m × n matrix C = ci j , where ci j = rai j , i = 1, 2, . . . , m and
j = 1, 2, . . . , n; that is, the matrix C is obtained by multiplying each entry of A
by r .

We have
EXAMPLE 10
4 −2 −3 (−2)(4) (−2)(−2) (−2)(−3)
−2 =
7 −3 2 (−2)(7) (−2)(−3) (−2)(2)
−8 4 6
= .
−14 6 −4 ■
Thus far, addition of matrices has been defined for only two matrices. Our
work with matrices will call for adding more than two matrices. Theorem 1.1
in Section 1.4 shows that addition of matrices satisfies the associative property:
A + (B + C) = (A + B) + C.
If A and B are m × n matrices, we write A + (−1)B as A − B and call this
the difference between A and B.
Let
EXAMPLE 11 2 3 −5 2 −1 3
A= and B= .
4 2 1 3 5 −2
Then
2−2 3 + 1 −5 − 3 0 4 −8
A−B = = .
4−3 2−5 1+2 1 −3 3 ■

Application
Vectors in R n can be used to handle large amounts of data. Indeed, a number of
computer software products, notably, M ATLAB R
, make extensive use of vectors.
The following example illustrates these ideas:
(Inventory Control) Suppose that a store handles 100 different items. The inven-
EXAMPLE 12
tory on hand at the beginning of the week can be described by the inventory vector
u in R 100 . The number of items sold at the end of the week can be described by
the 100-vector v, and the vector
u−v
represents the inventory at the end of the week. If the store receives a new shipment
of goods, represented by the 100-vector w, then its new inventory would be
u − v + w. ■

22
1.2 Matrices 17

We shall sometimes use the summation notation, and we now review this
useful and compact notation.
n
By ai we mean a1 + a2 + · · · + an . The letter i is called the index of
i=1
summation; it is a dummy variable that can be replaced by another letter. Hence
we can write
n n  n
ai = aj = ak .
i=1 j=1 k=1

Thus

4
ai = a1 + a2 + a3 + a4 .
i=1
The summation notation satisfies the following properties:

n 
n 
n
1. (ri + si )ai = ri ai + si ai
i=1 i=1 i=1
n 
n
2. c(ri ai ) = c ri ai
i=1 i=1
   

n 
m 
m 
n
3. ai j = ai j
j=1 i=1 i=1 j=1

Property 3 can be interpreted as follows: The left side is obtained by adding all the
entries in each column and then adding all the resulting numbers. The right side
is obtained by adding all the entries in each row and then adding all the resulting
numbers.
If A1 , A2 , . . . , Ak are m × n matrices and c1 , c2 , . . . , ck are real numbers, then
an expression of the form
c1 A1 + c2 A2 + · · · + ck Ak (2)
is called a linear combination of A1 , A2 , . . . , Ak , and c1 , c2 , . . . , ck are called
coefficients.
The linear combination in Equation (2) can also be expressed in summation
notation as
k
ci Ai = c1 A1 + c2 A2 + · · · + ck Ak .
i=1

The following are linear combinations of matrices:


EXAMPLE 13
⎡ ⎤ ⎡ ⎤
0 −3 5 5 2 3
1
3 ⎣2 3 4⎦ − ⎣ 6 2 3⎦ ,
2
1 −2 −3 −1 −2 3

2 3 −2 − 3 5 0 + 4 −2 5 ,
⎡ ⎤ ⎡ ⎤
1 0.1
−0.5 ⎣ −4 ⎦ + 0.4 ⎣ −4 ⎦ .
−6 0.2

23
18 Chapter 1 Linear Equations and Matrices

Using scalar multiplication and matrix addition, we can compute each of these
linear combinations. Verify that the results of such computations are, respectively,
⎡ ⎤ ⎡ ⎤
− 52 −10 27
2
−0.46
⎢ ⎥ ⎢ ⎥
⎢ 3 8 21 ⎥ , −17 16 , and ⎢ 0.4 ⎥ .
⎣ 2 ⎦ ⎣ ⎦
7 21
−5 − 2 3.08
2 ■
Let ⎡ ⎤
EXAMPLE 14 18.95
p = ⎣ 14.75 ⎦
8.60
be a 3-vector that represents the current prices of three items at a store. Suppose
that the store announces a sale so that the price of each item is reduced by 20%.
(a) Determine a 3-vector that gives the price changes for the three items.
(b) Determine a 3-vector that gives the new prices of the items.
Solution
(a) Since each item is reduced by 20%, the 3-vector
⎡ ⎤ ⎡ ⎤ ⎡ ⎤
(−0.20)18.95 −3.79 3.79
−0.20p = ⎣ (−0.20)14.75 ⎦ = ⎣ −2.95 ⎦ = − ⎣ 2.95 ⎦
(−0.20)8.60 −1.72 1.72
gives the price changes for the three items.
(b) The new prices of the items are given by the expression
⎡ ⎤ ⎡ ⎤ ⎡ ⎤
18.95 3.79 15.16
p − 0.20p = ⎣ 14.75 ⎦ − ⎣ 2.95 ⎦ = ⎣ 11.80 ⎦ .
8.60 1.72 6.88
Observe that this expression can also be written as
p − 0.20p = 0.80p. ■
The next operation on matrices is useful in a number of situations.
If A = ai j is an m × n matrix, then the transpose of A, A T = aiTj , is the
DEFINITION 1.5
n × m matrix defined by aiTj = a ji . Thus the transpose of A is obtained from A by
interchanging the rows and columns of A.

Let
EXAMPLE 15 ⎡ ⎤ ⎡ ⎤
6 2 −4 5 4
4 −2 3
A= , B = ⎣ 3 −1 2⎦ , C = ⎣ −3 2⎦ ,
0 5 −2
0 4 3 2 −3
⎡ ⎤
2
D = 3 −5 1 , E = ⎣ −1 ⎦ .
3

24
1.2 Matrices 19

Then
⎡ ⎤ ⎡ ⎤
4 0 6 3 0
A T = ⎣ −2 5⎦ , B T = ⎣ 2 −1 4⎦ ,
3 −2 −4 2 3
⎡ ⎤
3
5 −3 2
CT = , D T = ⎣ −5 ⎦ , and E T = 2 −1 3 .
4 2 −3
1 ■

Key Terms
Matrix Equal matrices Difference of matrices
Rows n-vector (or vector) Summation notation
Columns Rn , C n Index of summation
Size of a matrix 0, zero vector Linear combination
Square matrix Google Coefficients
Main diagonal Matrix addition Transpose
Element or entry of a matrix Scalar multiple

1.2 Exercises
⎡ ⎤
1. Let 0 1 0 0 0
⎡ ⎤
4 ⎢1 0
⎢ 1 1 1⎥⎥
2 −3 5
A= , B = ⎣ −3 ⎦ , (b) A = ⎢
⎢0 1 0 0 0⎥⎥
6 −5 4
5 ⎣0 1 0 0 0⎦
and 0 1 0 0 0
⎡ ⎤
7 3 2 4. If
C = ⎣ −4 3 5⎦. a+b c+d 4 6
= ,
6 1 −1 c−d a−b 10 2
(a) What is a12 , a22 , a23 ? find a, b, c, and d.
(b) What is b11 , b31 ? 5. If
(c) What is c13 , c31 , c33 ? a + 2b 2a − b 4 −2
= ,
2. Determine the incidence matrix associated with each of 2c + d c − 2d 4 −3
the following graphs: find a, b, c, and d.
(a) P3 (b) P2
In Exercises 6 through 9, let
P2
P3 ⎡ ⎤
P4 1 0
P1 P5 1 2 3
P1 A= , B = ⎣2 1⎦,
P5
P4 2 1 4
3 2
3. For each of the following incidence matrices, construct a ⎡ ⎤
graph. Label the vertices P1 , P2 , . . . , P5 . 3 −1 3
⎡ ⎤ 3 −2
0 1 0 1 1 C = ⎣4 1 5⎦, D = ,
2 4
⎢1 0 0 0 1⎥ 2 1 3
⎢ ⎥ ⎡ ⎤
(a) A = ⎢ ⎥ 2 −4
⎢0 0 0 1 1⎥ 5
−4 5
⎣1 0 1 0 1⎦ E = ⎣0 1 4⎦, F = ,
2 3
1 1 1 1 0 3 2 1

25
20 Chapter 1 Linear Equations and Matrices
⎡ ⎤
0 0 0 17. Show that the summation notation satisfies the following
and O = ⎣0 0 0⎦. properties:
0 0 0 n 
n 
n
(a) (ri + si )ai = ri ai + si ai
6. If possible, compute the indicated linear combination: i=1 i=1 i=1
 
(a) C + E and E + C (b) A + B 
n n
(b) c(ri ai ) = c ri ai
(c) D − F (d) −3C + 5O i=1 i=1
(e) 2C − 3E (f) 2B + F    

n m 
m 
n

7. If possible, compute the indicated linear combination: 18. Show that ai j = ai j .


i=1 j=1 j=1 i=1
(a) 3D + 2F (b) 3(2A) and 6A
19. Identify the following expressions as true or false. If true,
(c) 3A + 2A and 5A prove the result; if false, give a counterexample.
(d) 2(D + F) and 2D + 2F  
 n n
(e) (2 + 3)D and 2D + 3D (a) (ai + 1) = ai + n
i=1 i=1
(f) 3(B + D)  

n m
8. If possible, compute the following: (b) 1 = mn
(a) A T and (A T )T i=1 j=1
    
(b) (C + E)T and C T + E T 
m n n 
m
(c) ai b j = ai bj
(c) (2D + 3F)T (d) D − D T
j=1 i=1 i=1 j=1
T
(e) 2A + B (f) (3D − 2F)T 20. A large steel manufacturer, who has 2000 employees,
9. If possible, compute the following: lists each employee’s salary as a component of a vector
(a) (2A)T (b) (A − B)T u in R 2000 . If an 8% across-the-board salary increase has
been approved, find an expression involving u that gives
(c) (3B T − 2A)T (d) (3A T − 5B T )T
all the new salaries.
T T
(e) (−A) and −(A ) (f) (C + E + F T )T
21. A brokerage firm records the high and low values of the
3 0 price of IBM stock each day. The information for a given
10. Is the matrix a linear combination of the matri-
0 2 week is presented in two vectors, t and b, in R 5 , showing
1 0 1 0 the high and low values, respectively. What expression
ces and ? Justify your answer.
0 1 0 0 gives the average daily values of the price of IBM stock
for the entire 5-day week?
4 1
11. Is the matrix a linear combination of the ma- 22. For the software you are using, determine the commands
0−3
to enter a matrix, add matrices, multiply a scalar times a
1 0 1 0
trices and ? Justify your answer. matrix, and obtain the transpose of a matrix for matrices
0 1 0 0 with numerical entries. Practice the commands, using the
12. Let linear combinations in Example 13.
⎡ ⎤ ⎡ ⎤ 23. Determine whether the software you are using includes
1 2 3 1 0 0
A = ⎣6 −2 3⎦ and I3 = ⎣ 0 1 0⎦. a computer algebra system (CAS), and if it does, do the
following:
5 2 4 0 0 1
(a) Find the command for entering a symbolic matrix.
If λ is a real number, compute λI3 − A. (This command may be different than that for enter-
13. If A is an n × n matrix, what are the entries on the main ing a numeric matrix.)
diagonal of A − A T ? Justify your answer. (b) Enter several symbolic matrices like
14. Explain why every incidence matrix A associated with a
graph is the same as A T . r s t a b c
A= and B= .
u v w d e f
15. Let the n × n matrix A be equal to A T . Briefly describe
the pattern of the entries in A. Compute expressions like A + B, 2A, 3A + B,
16. If x is an n-vector, show that x + 0 = x. A − 2B, A T + B T , etc. (In some systems you must

26
1.3 Matrix Multiplication 21

explicitly indicate scalar multiplication with an as- matrix. If there is, display the graphs for the incidence
terisk.) matrices in Exercise 3 and compare them with those that
24. For the software you are using, determine whether there you drew by hand. Explain why the computer-generated
is a command that will display a graph for an incidence graphs need not be identical to those you drew by hand.

1.3 Matrix Multiplication


In this section we introduce the operation of matrix multiplication. Unlike ma-
trix addition, matrix multiplication has some properties that distinguish it from
multiplication of real numbers.
The dot product, or inner product, of the n-vectors in R n
DEFINITION 1.6
⎡ ⎤ ⎡ ⎤
a1 b1
⎢ a2 ⎥ ⎢ b2 ⎥
⎢ ⎥ ⎢ ⎥
a = ⎢ . ⎥ and b = ⎢ . ⎥
⎣ .. ⎦ ⎣ .. ⎦
an bn

is defined as

n
a · b = a1 b1 + a2 b2 + · · · + an bn = ai bi .∗
i=1

The dot product is an important operation that will be used here and in later
sections.
The dot product of
EXAMPLE 1 ⎡⎤ ⎡⎤
1 2
⎢ −2 ⎥ ⎢ 3⎥
u=⎢
⎣ 3⎦
⎥ and v=⎢ ⎥
⎣ −2 ⎦
4 1
is

u · v = (1)(2) + (−2)(3) + (3)(−2) + (4)(1) = −6. ■


⎡ ⎤ ⎡ ⎤
x 4
Let a = ⎣ 2 ⎦ and b = ⎣ 1 ⎦. If a · b = −4, find x.
EXAMPLE 2
3 2

Solution
We have
a · b = 4x + 2 + 6 = −4
4x + 8 = −4
x = −3. ■

∗ The dot product of vectors in C n is defined in Appendix B.2.

27
22 Chapter 1 Linear Equations and Matrices

(Computing a Course Average) Suppose that an instructor uses four grades to


EXAMPLE 3
determine a student’s course average: quizzes, two hourly exams, and a final exam.
These are weighted as 10%, 30%, 30%, and 30%, respectively. If a student’s scores
are 78, 84, 62, and 85, respectively, we can compute the course average by letting
⎡ ⎤ ⎡ ⎤
0.10 78
⎢ 0.30 ⎥ ⎢ 84 ⎥
w=⎢ ⎥ ⎢ ⎥
⎣ 0.30 ⎦ and g = ⎣ 62 ⎦
0.30 85
and computing
w · g = (0.10)(78) + (0.30)(84) + (0.30)(62) + (0.30)(85) = 77.1.
Thus the student’s course average is 77.1. ■

■ Matrix Multiplication
If A = ai j is an m × p matrix and B = bi j is a p × n matrix, then the product
DEFINITION 1.7
of A and B, denoted AB, is the m × n matrix C = ci j , defined by
ci j = ai1 b1 j + ai2 b2 j + · · · + ai p b pj
p
(1)
= aik bk j (1 ≤ i ≤ m, 1 ≤ j ≤ n).
k=1

Equation (1) says that the i, jth element in the product matrix is the dot prod-
uct of the transpose of the ith row, rowi (A)—that is, (rowi (A))T —and the jth
column, col j (B), of B; this is shown in Figure 1.4.

a11 a12 ... a1p colj (B)


a21 a22 ... a2p . . . b1j . . . b1n
b11 b12
.. .. .. . . . b2j . . . b2n
. . . b21 b22
rowi (A) ai1 ai2 . . . aip .. .. .. ..
. . . .
.. .. .. . . . bp j . . . bpn
. . . bp1 bp2
am1 am2 . . . amp
c11 c12 ... c1n
c21 c22 ... c2n
 .. .. ..
. cij
. .
cm1 cm2 . . . cmn
p

FIGURE 1.4
(rowi (A))T . colj (B)  兺a b
k1
ik kj  cij

Observe that the product of A and B is defined only when the number of rows
of B is exactly the same as the number of columns of A, as indicated in Figure 1.5.
Let ⎡ ⎤
EXAMPLE 4 −2 5
1 2 −1
A= and B = ⎣ 4 −3 ⎦ .
3 1 4
2 1

28
1.3 Matrix Multiplication 23

A B  AB
mp pn mn

the same

FIGURE 1.5 size of A B

Then
(1)(−2) + (2)(4) + (−1)(2) (1)(5) + (2)(−3) + (−1)(1)
AB =
(3)(−2) + (1)(4) + (4)(2) (3)(5) + (1)(−3) + (4)(1)
4 −2
= .
6 16 ■
Let ⎡ ⎤ ⎡ ⎤
EXAMPLE 5 1 −2 3 1 4
A = ⎣4 2 1⎦ and B = ⎣ 3 −1 ⎦ .
0 1 −2 −2 2
Compute the (3, 2) entry of AB.
Solution
If AB = C, then the (3, 2) entry of AB is c32 , which is (row3 (A))T · col2 (B). We
now have
⎡ ⎤ ⎡ ⎤
0 4
(row3 (A))T · col2 (B) = ⎣ 1 ⎦ · ⎣ −1 ⎦ = −5.
−2 2 ■
Let ⎡ ⎤
EXAMPLE 6 2
1 x 3
A= and B = ⎣ 4⎦ .
2 −1 1
y
12
If AB = , find x and y.
6

Solution
We have
⎡ ⎤
2
1 x 3 ⎣ ⎦ 2 + 4x + 3y 12
AB = 4 = = .
2 −1 1 4−4+y 6
y
Then
2 + 4x + 3y = 12
y = 6,
so x = −2 and y = 6. ■

29
24 Chapter 1 Linear Equations and Matrices

The basic properties of matrix multiplication will be considered in the next


section. However, multiplication of matrices requires much more care than their
addition, since the algebraic properties of matrix multiplication differ from those
satisfied by the real numbers. Part of the problem is due to the fact that AB is
defined only when the number of columns of A is the same as the number of rows
of B. Thus, if A is an m × p matrix and B is a p × n matrix, then AB is an m × n
matrix. What about B A? Four different situations may occur:
1. B A may not be defined; this will take place if n  = m.
2. If B A is defined, which means that m = n, then B A is p × p while AB is
m × m; thus, if m = p, AB and B A are of different sizes.
3. If AB and B A are both of the same size, they may be equal.
4. If AB and B A are both of the same size, they may be unequal.
If A is a 2 × 3 matrix and B is a 3 × 4 matrix, then AB is a 2 × 4 matrix while B A
EXAMPLE 7
is undefined. ■

Let A be 2 × 3 and let B be 3 × 2. Then AB is 2 × 2 while B A is 3 × 3. ■


EXAMPLE 8

Let
EXAMPLE 9 1 2 2 1
A= and B= .
−1 3 0 1
Then
2 3 1 7
AB = while BA = .
−2 2 −1 3
Thus AB = B A. ■
One might ask why matrix equality and matrix addition are defined in such
a natural way, while matrix multiplication appears to be much more complicated.
Only a thorough understanding of the composition of functions and the relation-
ship that exists between matrices and what are called linear transformations would
show that the definition of multiplication given previously is the natural one. These
topics are covered later in the book. For now, Example 10 provides a motivation
for the definition of matrix multiplication.
(Ecology) Pesticides are sprayed on plants to eliminate harmful insects. However,
EXAMPLE 10
some of the pesticide is absorbed by the plant. The pesticides are absorbed by her-
bivores when they eat the plants that have been sprayed. To determine the amount
of pesticide absorbed by a herbivore, we proceed as follows. Suppose that we have
three pesticides and four plants. Let ai j denote the amount of pesticide i (in mil-
ligrams) that has been absorbed by plant j. This information can be represented
by the matrix

⎡ Plant 1 Plant 2 Plant 3 Plant 4 ⎤


2 3 4 3 Pesticide 1
A= ⎣ 3 2 2 5 ⎦ Pesticide 2 .
4 1 6 4 Pesticide 3

30
1.3 Matrix Multiplication 25

Now suppose that we have three herbivores, and let bi j denote the number of plants
of type i that a herbivore of type j eats per month. This information can be repre-
sented by the matrix
Herbivore 1 Herbivore 2 Herbivore 3
⎡ ⎤
20 12 8 Plant 1
⎢ 28 15 15 ⎥ Plant 2
B= ⎣ ⎦ Plant 3 .
30 12 10
40 16 20 Plant 4

The (i, j) entry in AB gives the amount of pesticide of type i that animal j has
absorbed. Thus, if i = 2 and j = 3, the (2, 3) entry in AB is
(row2 (A))T · col3 (B) = 3(8) + 2(15) + 2(10) + 5(20)
= 174 mg of pesticide 2 absorbed by herbivore 3.
If we now have p carnivores (such as a human) who eat the herbivores, we can
repeat the analysis to find out how much of each pesticide has been absorbed by
each carnivore. ■

It is sometimes useful to be able to find a column in the matrix product AB


without having to multiply the two matrices. It is not difficult to show (Exercise
46) that the jth column of the matrix product AB is equal to the matrix product
Acol j (B).
Let ⎡ ⎤
EXAMPLE 11 1 2
−2 3 4
A=⎣ 3 4⎦ and B= .
3 2 1
−1 5
Then the second column of AB is
⎡ ⎤ ⎡ ⎤
1 2 7
3
Acol2 (B) = ⎣ 3 4⎦ = ⎣ 17 ⎦ .
2
−1 5 7 ■

Remark If u and v are n-vectors (n × 1 matrices), then it is easy to show by


matrix multiplication (Exercise 41) that
u · v = uT v.
This observation is applied in Chapter 5.

■ The Matrix–Vector Product Written in Terms of Columns


Let ⎡ ⎤
a11 a12 ··· a1n
⎢ a21 a22 ··· a2n ⎥
⎢ ⎥
A=⎢ . .. .. ⎥
⎣ .. . . ⎦
am1 am2 · · · amn

31
26 Chapter 1 Linear Equations and Matrices

be an m × n matrix and let ⎡ ⎤


c1
⎢ c2 ⎥
⎢ ⎥
c=⎢ . ⎥
⎣ .. ⎦
cn
be an n-vector, that is, an n × 1 matrix. Since A is m × n and c is n × 1, the matrix
product Ac is the m × 1 matrix
⎡ ⎤⎡ ⎤ ⎡ ⎤
a11 a12 · · · a1n c1 (row1 (A))T · c
⎢ a21 a22 · · · a2n ⎥ ⎢ c2 ⎥ ⎢ (row2 (A))T · c ⎥
⎢ ⎥⎢ ⎥ ⎢ ⎥
Ac = ⎢ . .. .. ⎥ ⎢ .. ⎥ = ⎢ .. ⎥
⎣ . . . . ⎦⎣ . ⎦ ⎣ . ⎦
am1 am2 · · · amn cn (rowm (A))T · c
⎡ ⎤ (2)
a11 c1 + a12 c2 + · · · + a1n cn
⎢ a21 c1 + a22 c2 + · · · + a2n cn ⎥
⎢ ⎥
=⎢ .. ⎥.
⎣ . ⎦
am1 c1 + am2 c2 + · · · + amn cn
This last expression can be written as
⎡ ⎤ ⎡ ⎤ ⎡ ⎤
a11 a12 a1n
⎢ a21 ⎥ ⎢ a22 ⎥ ⎢ a2n ⎥
⎢ ⎥ ⎢ ⎥ ⎢ ⎥
c1 ⎢ . ⎥ + c2 ⎢ . ⎥ + · · · + cn ⎢ . ⎥
.
⎣ . ⎦ .
⎣ . ⎦ .
⎣ . ⎦ (3)
am1 am2 amn

= c1 col1 (A) + c2 col2 (A) + · · · + cn coln (A).


Thus the product Ac of an m × n matrix A and an n × 1 matrix c can be written as
a linear combination of the columns of A, where the coefficients are the entries in
the matrix c.
In our study of linear systems of equations we shall see that these systems can
be expressed in terms of a matrix–vector product. This point of view provides us
with an important way to think about solutions of linear systems.
Let ⎡ ⎤
EXAMPLE 12 2
2 −1 −3
A= and c = ⎣ −3 ⎦ .
4 2 −2
4
Then the product Ac, written as a linear combination of the columns of A, is
⎡ ⎤
2
2 −1 −3 ⎣ 2 −1 −3 −5
Ac = −3 ⎦ = 2 −3 +4 = .
4 2 −2 4 2 −2 −6
4 ■
If A is an m × p matrix and B is a p × n matrix, we can then conclude that
the jth column of the product AB can be written as a linear combination of the

32
1.3 Matrix Multiplication 27

columns of matrix A, where the coefficients are the entries in the jth column of
matrix B:

col j (AB) = Acol j (B) = b1 j col1 (A) + b2 j col2 (A) + · · · + b pj col p (A).

If A and B are the matrices defined in Example 11, then


EXAMPLE 13
⎡ ⎤ ⎡ ⎤
1 2 4 7 6
−2 3 4
AB = ⎣ 3 4⎦ =⎣ 6 17 16 ⎦ .
3 2 1
−1 5 17 7 1

The columns of AB as linear combinations of the columns of A are given by

⎡⎤ ⎡ ⎤ ⎡ ⎤
4 1 2
col1 (AB) = ⎣ 6 ⎦ = Acol1 (B) = −2 ⎣ 3 ⎦ + 3 ⎣ 4 ⎦
17 −1 5
⎡ ⎤ ⎡ ⎤ ⎡ ⎤
7 1 2
⎣ ⎦
col2 (AB) = 17 = Acol2 (B) = 3 ⎣ 3 + 2 4⎦
⎦ ⎣
7 −1 5
⎡ ⎤ ⎡ ⎤ ⎡ ⎤
6 1 2
⎣ ⎦
col3 (AB) = 16 = Acol3 (B) = 4 ⎣ 3 + 1 4⎦ .
⎦ ⎣
1 −1 5 ■

■ Linear Systems

Consider the linear system of m equations in n unknowns,

a11 x1 + a12 x2 + · · · + a1n xn = b1


a21 x1 + a22 x2 + · · · + a2n xn = b2
.. .. .. .. (4)
. . . .
am1 x1 + am2 x2 + · · · + amn xn = bm .

Now define the following matrices:

⎡ ⎤ ⎡ ⎤ ⎡ ⎤
a11 a12 ··· a1n x1 b1
⎢ a21 a22 ··· a2n ⎥ ⎢ x2 ⎥ ⎢ b2 ⎥
⎢ ⎥ ⎢ ⎥ ⎢ ⎥
A=⎢ . .. .. ⎥ , x = ⎢ . ⎥, b = ⎢ . ⎥.
⎣ .. . . ⎦ ⎣ .. ⎦ ⎣ .. ⎦
am1 am2 · · · amn xn bm

33
28 Chapter 1 Linear Equations and Matrices

Then
⎡ ⎤⎡ ⎤
a11 a12 ··· a1n x1
⎢ a21 a22 ··· a2n ⎥ ⎢ x2 ⎥
⎢ ⎥⎢ ⎥
Ax = ⎢ . .. .. ⎥ ⎢ .. ⎥
⎣ .. . . ⎦⎣ . ⎦
am1 am2 · · · amn xn
⎡ ⎤ (5)
a11 x1 + a12 x2 + · · · + a1n xn
⎢ a21 x1 + a22 x2 + · · · + a2n xn ⎥
=⎢
⎣ ... .. .. ⎥ .
. . ⎦
am1 x1 + am2 x2 + · · · + amn xn
The entries in the product Ax at the end of (5) are merely the left sides of the
equations in (4). Hence the linear system (4) can be written in matrix form as
Ax = b.
The matrix A is called the coefficient matrix of the linear system (4), and the
matrix ⎡ ⎤
a11 a12 · · · a1n b1
⎢ a21 a22 · · · a2n b2 ⎥
⎢ ⎥
⎢ .. .. .. .. ⎥ ,
⎣ . . . . ⎦
am1 am2 · · · amn bm
obtained by adjoining column b to A, is called the augmented matrix of the linear
system (4). The augmented matrix of (4) is written as A b . Conversely, any
matrix with more than one column can be thought of as the augmented matrix
of a linear system. The coefficient and augmented matrices play key roles in our
method for solving linear systems.
Recall from Section 1.1 that if
b1 = b2 = · · · = bm = 0
in (4), the linear system is called a homogeneous system. A homogeneous system
can be written as
Ax = 0,
where A is the coefficient matrix.
Consider the linear system
EXAMPLE 14
−2x + z=5
2x + 3y − 4z = 7
3x + 2y + 2z = 3.

Letting
⎡ ⎤ ⎡ ⎤ ⎡ ⎤
−2 0 1 x 5
A= ⎣ 2 3 −4 ⎦ , x = y⎦,
⎣ and b = 7⎦ ,

3 2 2 z 3

34
1.3 Matrix Multiplication 29

we can write the given linear system in matrix form as


Ax = b.
The coefficient matrix is A, and the augmented matrix is
⎡ ⎤
−2 0 1 5
⎣ 2 3 −4 7⎦ .
3 2 2 3 ■
The matrix
EXAMPLE 15 2 −1 3 4
3 0 2 5
is the augmented matrix of the linear system
2x − y + 3z = 4
3x + 2z = 5. ■
We can express (5) in another form, as follows, using (2) and (3):
⎡ ⎤
a11 x1 + a12 x2 + · · · + a1n xn
⎢ a21 x1 + a22 x2 + · · · + a2n xn ⎥
⎢ ⎥
Ax = ⎢ .. ⎥
⎣ . ⎦
am1 x1 + am2 x2 + · · · + amn xn
⎡ ⎤ ⎡ ⎤ ⎡ ⎤
a11 x1 a12 x2 a1n xn
⎢ a21 x1 ⎥ ⎢ a22 x2 ⎥ ⎢ a2n xn ⎥
⎢ ⎥ ⎢ ⎥ ⎢ ⎥
= ⎢ . ⎥ + ⎢ . ⎥ + ··· + ⎢ . ⎥
⎣ .. ⎦ ⎣ .. ⎦ ⎣ .. ⎦
am1 x1 am2 x2 amn xn
⎡ ⎤ ⎡ ⎤ ⎡ ⎤
a11 a12 a1n
⎢ a21 ⎥ ⎢ a22 ⎥ ⎢ a2n ⎥
⎢ ⎥ ⎢ ⎥ ⎢ ⎥
= x1 ⎢ . ⎥ + x2 ⎢ . ⎥ + · · · + xn ⎢ . ⎥
⎣ .. ⎦ ⎣ .. ⎦ ⎣ .. ⎦
am1 am2 amn
= x1 col1 (A) + x2 col2 (A) + · · · + xn coln (A).
Thus Ax is a linear combination of the columns of A with coefficients that are the
entries of x. It follows that the matrix form of a linear system, Ax = b, can be
expressed as
x1 col1 (A) + x2 col2 (A) + · · · + xn coln (A) = b. (6)
Conversely, an equation of the form in (6) always describes a linear system of the
form in (4).
Consider the linear system Ax = b, where the coefficient matrix
EXAMPLE 16
⎡ ⎤ ⎡ ⎤
3 1 2 ⎡ ⎤ 4
⎢ 4 −5 ⎥ x1 ⎢1⎥
6
A=⎢ ⎣ 0
⎥ , x = ⎣ x2 ⎦ , and b = ⎢ ⎥ .
7 −3 ⎦ ⎣0⎦
x3
−1 2 0 2

35
30 Chapter 1 Linear Equations and Matrices

Writing Ax = b as a linear combination of the columns of A as in (6), we have


⎡ ⎤ ⎡ ⎤ ⎡ ⎤ ⎡ ⎤
3 1 2 4
⎢ 4⎥ ⎢ −5 ⎥ ⎢ 6⎥ ⎢1⎥
x1 ⎢ ⎥ ⎢ ⎥ ⎢ ⎥ ⎢ ⎥
⎣ 0 ⎦ + x2 ⎣ 7 ⎦ + x3 ⎣ −3 ⎦ = ⎣ 0 ⎦ .
−1 2 0 2 ■
The expression for the linear system Ax = b as shown in (6), provides an
important way to think about solutions of linear systems.

Ax = b is consistent if and only if b can be expressed as a linear combination


of the columns of the matrix A.

We encounter this approach in Chapter 2.

Key Terms
Dot product (inner product) Coefficient matrix
Matrix–vector product Augmented matrix

1.3 Exercises
⎡ ⎤
In Exercises 1 and 2, compute a · b. −3
3. Let a = b = ⎣ 2 ⎦. If a · b = 17, find x.
1 4
1. (a) a = ,b= x
2 −1
4. Determine the value of x so that v · w = 0, where
−3 1 ⎡ ⎤ ⎡ ⎤
(b) a = ,b= 1 x
−2 −2 ⎢ −3 ⎥ ⎢ 2⎥
v=⎢ ⎥ ⎢ ⎥
⎡ ⎤ ⎡ ⎤ ⎣ 4 ⎦ and w = ⎣ −1 ⎦ .
4 1
x 1
(c) a = ⎣ 2 ⎦, b = ⎣ 3 ⎦
−1 6 5. Determine values
⎡ ⎤ ⎤ v · w = 0⎡and
of x and⎡y so that ⎤v · u =
⎡ ⎤ ⎡ ⎤ x 2 1
1 1 0, where v = ⎣ 1 ⎦, w = ⎣ −2 ⎦, and u = ⎣ 8 ⎦.
(d) a = ⎣ 1 ⎦, b = ⎣ 0 ⎦ y 1 2
0 1 6. Determine values of x and y so that v · w = 0 and v·u =
⎡ ⎤ ⎡ ⎤ ⎡ ⎤
x x 0
2 3
2. (a) a = ,b= 0, where v = ⎣ 1 ⎦, w = ⎣ −2 ⎦, and u = ⎣ −9 ⎦.
−1 2
y 0 y
1 1 sin θ
(b) a = ,b= 7. Let w = . Compute w · w.
−1 1 cos θ
⎡ ⎤ ⎡ ⎤ ⎡ ⎤
1 −2 x
(c) a = ⎣ 2 ⎦, b = ⎣ 0 ⎦ 8. Find all values of x so that u · u = 50, where u = ⎣ 3 ⎦.
3 1 4
⎡ 1⎤
⎡ ⎤ ⎡ ⎤
1 1 ⎢
2

(d) a = ⎣ 0 ⎦, b = ⎣ 0 ⎦ 9. Find all values of x so that v · v = 1, where v = ⎣ − 1 ⎦.
2
0 0 x

36
1.3 Matrix Multiplication 31
⎡ ⎤ ⎡ ⎤
y 2 3
1 2 x 3 −1 3
10. Let A = and B = ⎣ x ⎦. 17. Let A = ⎣ −1 4 ⎦ and B = .
3 −1 2 1 2 4
1 0 3
6 Compute the following entries of AB:
If AB = , find x and y.
8 (a) the (1, 2) entry (b) the (2, 3) entry
Consider the following matrices for Exercises 11 through 15: (c) the (3, 1) entry (d) the (3, 3) entry
⎡ ⎤ 1 0 2 3
1 0 18. If I2 = and D = , compute D I2
1 2 3 0 1 −1 −2
A= , B = ⎣2 1⎦, and I2 D.
2 1 4
3 2 19. Let
⎡ ⎤
3 −1 3 1 2 2 −1
3 −2 A= and B= .
C = ⎣4 1 5⎦, D= , 3 2 −3 4
2 5
2 1 3
Show that AB  = B A.
⎡ ⎤ ⎡ ⎤
2 −4 5 −1 2 20. If A is the matrix in Example 4 and O is the 3 × 2 matrix
E = ⎣0 1 4⎦, and F =⎣ 0 4⎦. every one of whose entries is zero, compute AO.
3 2 1 3 5
In Exercises 21 and 22, let
11. If possible, compute the following: ⎡ ⎤
1 −1 2
(a) AB (b) B A (c) F T E ⎢3 2 4⎥
A=⎢ ⎥
(d) C B + D (e) AB + D 2 , where D 2 = D D ⎣4 −2 3⎦
12. If possible, compute the following: 2 1 5
(a) D A + B (b) EC (c) C E and ⎡ ⎤
(d) E B + F (e) FC + D 1 0 −1 2
13. If possible, compute the following: B = ⎣3 3 −3 4⎦.
4 2 5 1
(a) F D − 3B (b) AB − 2D
21. Using the method in Example 11, compute the following
(c) F T B + D (d) 2F − 3(AE)
columns of AB:
(e) B D + AE (a) the first column (b) the third column
14. If possible, compute the following: 22. Using the method in Example 11, compute the following
(a) A(B D) (b) (AB)D columns of AB:
(c) A(C + E) (d) AC + AE (a) the second column (b) the fourth column
T T
(e) (2AB) and 2(AB) (f) A(C − 3E) 23. Let
15. If possible, compute the following: ⎡ ⎤ ⎡ ⎤
2 −3 4 2
(a) A T (b) (A T )T A = ⎣ −1 2 3 ⎦ and c = ⎣ 1 ⎦ .
(c) (AB)T (d) B T A T 5 −1 −2 4
(e) (C + E)T B and C T B + E T B Express Ac as a linear combination of the columns of A.
(f) A(2B) and 2(AB) 24. Let
16. Let A = 1 2 −3 , B = −1 4 2 , and ⎡ ⎤ ⎡ ⎤
1 −2 −1 1 −1
C = −3 0 1 . If possible, compute the following:
A = ⎣2 4 3⎦ and B = ⎣3 2⎦.
(a) AB T (b) C A T (c) (B A T )C 3 0 −2 2 4
(d) A T B (e) CC T (f) C T C
Express the columns of AB as linear combinations of the
T T
(g) B C A A columns of A.

37
32 Chapter 1 Linear Equations and Matrices
⎡ ⎤
3 33. Write the following linear system in matrix form:
2 −3 1
25. Let A = and B = ⎣ 5 ⎦.
1 2 4 2x1 + 3x2 = 0
2
3x2 + x3 = 0
(a) Verify that AB = 3a1 + 5a2 + 2a3 , where a j is the
jth column of A for j = 1, 2, 3. 2x1 − x2 = 0
(row1 (A))B 34. Write the linear system whose augmented matrix is
(b) Verify that AB = . ⎡ ⎤
(row2 (A))B 2 1 3 4 0
26. (a) Find a value of r so that AB T = 0, where (a) ⎣ 3 −1 2 0 3 ⎦.
A = r 1 −2 and B = 1 3 −1 . −2 1 −4 3 2
⎡ ⎤
(b) Give an alternative way to write this product. 2 1 3 4 0
27. Find a value of r and a value of s so that AB T = 0, where ⎢ 3 −1 2 0 3⎥
(b) ⎢
⎣ −2
⎥.
A = 1 r 1 and B = −2 2 s . 1 −4 3 2⎦
28. (a) Let A be an m × n matrix with a row consisting en- 0 0 0 0 0
tirely of zeros. Show that if B is an n × p matrix, 35. How are the linear systems obtained in Exercise 34 re-
then AB has a row of zeros. lated?
(b) Let A be an m × n matrix with a column consisting 36. Write each of the following linear systems as a linear
entirely of zeros and let B be p × m. Show that B A combination of the columns of the coefficient matrix:
has a column of zeros. (a) 3x1 + 2x2 + x3 = 4
−3 2 1 x1 − x2 + 4x3 = −2
29. Let A = with a j = the jth column of A,
4 5 0 (b) −x1 + x2 = 3
j = 1, 2, 3. Verify that 2x1 − x2 = −2
⎡ ⎤ 3x1 + x2 = 1
a1T a1 a1T a2 a1T a3 37. Write each of the following linear combinations of
⎢ ⎥
AT A = ⎢ T
⎣ a2 a1 a2T a2 a2T a3 ⎥
⎦. columns as a linear system of the form in (4):
a3T a1 a3T a2 a3T a3 2 1 4
(a) x1 + x2 =
0 3 2
30. Consider the following linear system: ⎡ ⎤ ⎡ ⎤ ⎡ ⎤ ⎡ ⎤ ⎡ ⎤
1 0 3 1 2
(b) x1 ⎣ 2 ⎦ + x 2 ⎣ 1 ⎦ + x 3 ⎣ 4 ⎦ + x 4 ⎣ 3 ⎦ = ⎣ 5 ⎦
2x1 + 3x2 − 3x3 + x4 + x5 = 7
−1 2 5 4 8
3x1 + 2x3 + 3x5 = −2
38. Write each of the following as a linear system in matrix
2x1 + 3x2 − 4x4 = 3 form:
x3 + x4 + x5 = 5. 1 2 0 1
(a) x1 + x2 + x3 =
(a) Find the coefficient matrix. 2 5 3 1
⎡ ⎤ ⎡ ⎤ ⎡ ⎤ ⎡ ⎤
(b) Write the linear system in matrix form. 1 2 1 0
(b) x1 ⎣ 1 ⎦ + x2 ⎣ 1 ⎦ + x3 ⎣ 2 ⎦ = ⎣ 0 ⎦
(c) Find the augmented matrix.
2 0 2 0
31. Write the linear system whose augmented matrix is
39. Determine a solution to each of the following linear sys-
⎡ ⎤ tems, using the fact that Ax = b is consistent if and only
−2 −1 0 4 5
⎢ −3 2 7 8 3⎥ if b is a linear combination of the columns of A:
⎢ ⎥. ⎡ ⎤⎡ ⎤ ⎡ ⎤
⎣ 1 0 0 2 4⎦ 1 2 1 x1 0
3 0 1 3 6 (a) ⎣ −3 6 −3 ⎦ ⎣ x2 ⎦ = ⎣ 0 ⎦
0 1 −1 x3 0
32. Write the following linear system in matrix form: ⎡ ⎤
⎡ ⎤ x ⎡ ⎤
1 2 3 4 ⎢ 1⎥ 20
x
−2x1 + 3x2 = 5 (b) ⎣ 2 3 4 1 ⎦ ⎢ 2 ⎥ ⎣ ⎦
⎣ x3 ⎦ = 20
x1 − 5x2 = 4 3 4 1 2 20
x4

38
1.3 Matrix Multiplication 33
⎡ ⎤
1 47. Show that the jth column of the matrix product AB is a
40. Construct a coefficient matrix A so that x = ⎣ 0 ⎦ is a linear combination of the columns of A with coefficients
1 the entries in b j , the jth column of B.
⎡ ⎤
1 48. The vector ⎡ ⎤
solution to the system Ax = b, where b = ⎣ 2 ⎦. Can 20
3 ⎢ 30 ⎥
u=⎢ ⎥
there be more than one such coefficient matrix? Explain. ⎣ 80 ⎦
41. Show that if u and v are n-vectors, then u · v = uT v. 10
42. Let A be an m ×n matrix and B an n × p matrix. What, if gives the number of receivers, CD players, speakers, and
anything, can you say about the matrix product AB when DVD recorders that are on hand in an audio shop. The
vector ⎡ ⎤
(a) A has a column consisting entirely of zeros? 200
(b) B has a row consisting entirely of zeros? ⎢ 120 ⎥
v=⎢ ⎣ 80 ⎦

43. If A = ai j is an n × n matrix, then the trace of A,
Tr(A), is defined as the sum of all elements on the main 70
n
diagonal of A, Tr(A) = aii . Show each of the follow- gives the price (in dollars) of each receiver, CD player,
i=1 speaker, and DVD recorder, respectively. What does the
ing:
dot product u · v tell the shop owner?
(a) Tr(c A) = c Tr(A), where c is a real number
49. (Manufacturing Costs) A furniture manufacturer
(b) Tr(A + B) = Tr(A) + Tr(B) makes chairs and tables, each of which must go through
(c) Tr(AB) = Tr(B A) an assembly process and a finishing process. The times
(d) Tr(A T ) = Tr(A) required for these processes are given (in hours) by the
(e) Tr(A T A) ≥ 0 matrix
Assembly Finishing
44. Compute the trace (see Exercise 43) of each of the fol- process process
lowing matrices: 2 2 Chair
⎡ ⎤ A= .
2 2 3 3 4 Table
1 0
(a) (b) ⎣ 2 4 4⎦ The manufacturer has a plant in Salt Lake City and an-
2 3
3 −2 −5 other in Chicago. The hourly rates for each of the pro-
⎡ ⎤ cesses are given (in dollars) by the matrix
1 0 0
Salt Lake
(c) ⎣ 0 1 0 ⎦ City Chicago
0 0 1 9 10 Assembly process
45. Show that there are no 2 × 2 matrices A and B such that B= .
10 12 Finishing process
1 0
AB − B A = . What do the entries in the matrix product AB tell the
0 1
manufacturer?
46. (a) Show that the jth column of the matrix product AB
is equal to the matrix product Ab j , where b j is the 50. (Medicine) A diet research project includes adults and
jth column of B. It follows that the product AB can children of both sexes. The composition of the partici-
be written in terms of columns as pants in the project is given by the matrix

AB = Ab1 Ab2 ··· Abn . Adults Children


80 120 Male
(b) Show that the ith row of the matrix product AB is A= .
100 200 Female
equal to the matrix product ai B, where ai is the ith
row of A. It follows that the product AB can be The number of daily grams of protein, fat, and carbohy-
written in terms of rows as drate consumed by each child and adult is given by the
⎡ ⎤ matrix
a1 B
Carbo-
⎢ a2 B ⎥ Protein Fat hydrate
⎢ ⎥
AB = ⎢ . ⎥ .
⎣ .. ⎦ B=
20 20 20 Adult
.
10 20 30 Child
am B

39
34 Chapter 1 Linear Equations and Matrices

(a) How many grams of protein are consumed daily by (c) Show that (ka) · b = a · (kb) = k(a · b).
the males in the project? 53. Let A be an m ×n matrix whose entries are real numbers.
(b) How many grams of fat are consumed daily by the Show that if A A T = O (the m × m matrix all of whose
females in the project? entries are zero), then A = O.
51. Let x be an n-vector. 54. Use the matrices A and C in Exercise 11 and the ma-
trix multiplication command in your software to compute
(a) Is it possible for x · x to be negative? Explain.
AC and C A. Discuss the results.
(b) If x · x = 0, what is x? 55. Using your software, compute B T B and B B T for
52. Let a, b, and c be n-vectors and let k be a real number.
1 1 1 1 1
(a) Show that a · b = b · a. B= 1 2 3 4 5 6
.

(b) Show that (a + b) · c = a · c + b · c. Discuss the nature of the results.

1.4 Algebraic Properties of Matrix Operations


In this section we consider the algebraic properties of the matrix operations just
defined. Many of these properties are similar to the familiar properties that hold
for real numbers. However, there will be striking differences between the set
of real numbers and the set of matrices in their algebraic behavior under certain
operations—for example, under multiplication (as seen in Section 1.3). The proofs
of most of the properties will be left as exercises.

Theorem 1.1 Properties of Matrix Addition


Let A, B, and C be m × n matrices.
(a) A + B = B + A.
(b) A + (B + C) = (A + B) + C.
(c) There is a unique m × n matrix O such that
A+O = A (1)
for any m × n matrix A. The matrix O is called the m × n zero matrix.
(d) For each m × n matrix A, there is a unique m × n matrix D such that
A + D = O. (2)
We shall write D as −A, so (2) can be written as
A + (−A) = O.
The matrix −A is called the negative of A. We also note that −A is (−1)A.
Proof
(a) Let
A = ai j , B = bi j ,
A + B = C = ci j , and B + A = D = di j .
We must show that ci j = di j for all i, j. Now ci j = ai j + bi j and di j = bi j + ai j
for all i, j. Since ai j and bi j are real numbers, we have ai j + bi j = bi j + ai j , which
implies that ci j = di j for all i, j.

40
1.4 Algebraic Properties of Matrix Operations 35

(c) Let U = u i j . Then A + U = A if and only if† ai j + u i j = ai j , which holds


if and only if u i j = 0. Thus U is the m × n matrix all of whose entries are zero: U
is denoted by O. ■
The 2 × 2 zero matrix is
0 0
O= .
0 0
If
EXAMPLE 1 4 −1
A= ,
2 3
then
4 −1 0 0 4 + 0 −1 + 0 4 −1
+ = = .
2 3 0 0 2+0 3+0 2 3 ■
The 2 × 3 zero matrix is
0 0 0
O= .
0 0 0

1 3 −2 −1 −3 2
If A = , then −A = . ■
EXAMPLE 2 −2 4 3 2 −4 −3

Theorem 1.2 Properties of Matrix Multiplication


(a) If A, B, and C are matrices of the appropriate sizes, then
A(BC) = (AB)C.
(b) If A, B, and C are matrices of the appropriate sizes, then
(A + B)C = AC + BC.
(c) If A, B, and C are matrices of the appropriate sizes, then
C(A + B) = C A + C B. (3)
Proof
(a) Suppose that A is m × n, B is n × p, and C is p × q. We shall prove the
result for the special case m = 2, n = 3, p = 4, and q = 3. The general proof is
completely analogous.
Let A = ai j , B = bi j , C = ci j , AB = D = di j , BC = E = ei j ,
(AB)C = F = f i j , and A(BC) = G = gi j . We must show that f i j = gi j for
all i, j. Now
 

4 
4 3
fi j = dik ck j = air br k ck j
k=1 k=1 r =1


The connector “if and only if” means that both statements are true or both statements are false.
Thus (i) if A + U = A, then ai j + u i j = ai j ; and (ii) if ai j + u i j = ai j , then A + U = A. See Appendix
C, “Introduction to Proofs.”

41
36 Chapter 1 Linear Equations and Matrices

and
 

3 
3 
4
gi j = air er j = air br k ck j .
r =1 r =1 k=1

Then, by the properties satisfied by the summation notation,



4
fi j = (ai1 b1k + ai2 b2k + ai3 b3k )ck j
k=1

4 
4 
4
= ai1 b1k ck j + ai2 b2k ck j + ai3 b3k ck j
k=1 k=1 k=1
 

3 4
= air br k ck j = gi j .
r =1 k=1

The proofs of (b) and (c) are left as Exercise 4. ■


Let ⎡ ⎤
EXAMPLE 3 2 −1 1 0
5 2 3
A= , ⎣
B= 0 2 2 2⎦ ,
2 −3 4
3 0 −1 3
and ⎡ ⎤
1 0 2
⎢ 2 −3 0⎥
C =⎢
⎣0
⎥.
0 3⎦
2 1 0
Then
⎡ ⎤
0 3 7
5 2 3 ⎣ 43 16 56
A(BC) = 8 −4 6⎦ =
2 −3 4 12 30 8
9 3 3
and
⎡ ⎤
1 0 2
19 −1 6 ⎢
13 ⎢ 2 −3 0⎥
(AB)C = ⎥ = 43 16 56
.
16 −8 −8 6 ⎣0 0 3⎦ 12 30 8 ■
2 1 0
Let
EXAMPLE 4 ⎤

1 0
2 2 3 0 0 1
A= , B= , and C = ⎣ 2 2⎦ .
3 −1 2 2 3 −1
3 −1
Then ⎤ ⎡
1 0
2 2 4 ⎣ 18 0
(A + B)C = 2 2⎦ =
5 2 1 12 3
3 −1

42
1.4 Algebraic Properties of Matrix Operations 37

and (verify)
15 1 3 −1 18 0
AC + BC = + = .
7 −4 5 7 12 3 ■

Recall Example 9 in Section 1.3, which shows that AB need not always
equal B A. This is the first significant difference between multiplication of
matrices and multiplication of real numbers.

Theorem 1.3 Properties of Scalar Multiplication


If r and s are real numbers and A and B are matrices of the appropriate sizes, then
(a) r (s A) = (r s)A
(b) (r + s)A = r A + s A
(c) r (A + B) = r A + r B
(d) A(r B) = r (AB) = (r A)B
Proof
Exercises 13, 14, 16, and 18. ■
Let ⎡ ⎤
EXAMPLE 5 3 −2 1
4 2 3
A= and B = ⎣2 0 −1 ⎦ .
2 −3 4
0 1 2
Then
12 6 9 24 12 18
2(3A) = 2 = = 6A.
6 −9 12 12 −18 24
We also have
⎡ ⎤
6 −4 2
4 2 3 ⎣ 32 −10 16
A(2B) = 4 0 −2 ⎦ = = 2(AB).
2 −3 4 0 0 26
0 2 4 ■
Scalar multiplication can be used to change the size of entries in a matrix to meet
EXAMPLE 6
prescribed properties. Let ⎡ ⎤
3
⎢7⎥
A=⎢ ⎥
⎣2⎦ .
1
Then for k = 17 , the largest entry of k A is 1. Also if the entries of A represent the
volume of products in gallons, for k = 4, k A gives the volume in quarts. ■
So far we have seen that multiplication and addition of matrices have much in
common with multiplication and addition of real numbers. We now look at some
properties of the transpose.

43
38 Chapter 1 Linear Equations and Matrices

Theorem 1.4 Properties of Transpose


If r is a scalar and A and B are matrices of the appropriate sizes, then
(a) (A T )T = A
(b) (A + B)T = A T + B T
(c) (AB)T = B T A T
(d) (r A)T = r A T
Proof
We leave the proofs of (a), (b), and (d) as Exercises 26 and 27.
(c) Let A = ai j and B = bi j ; let AB = C = ci j . We must prove that ciTj
is the (i, j) entry in B T A T . Now

n 
n
ciTj = c ji = a jk bki = akTj bik
T

k=1 k=1


n
T T
= bik ak j = the (i, j) entry in B T A T . ■
k=1

Let
EXAMPLE 7
1 2 3 3 −1 2
A= and B= .
−2 0 1 3 2 −1
Then ⎡ ⎤ ⎡ ⎤
1 −2 3 3
AT = ⎣ 2 0⎦ and B T = ⎣ −1 2⎦ .
3 1 2 −1
Also,
⎡ ⎤
4 1
4 1 5
A+B = and (A + B)T = ⎣ 1 2 ⎦ .
1 2 0
5 0
Now
⎡⎤
4 1
A T + B T = ⎣ 1 2 ⎦ = (A + B)T .
5 0 ■
Let ⎡ ⎤
EXAMPLE 8
0 1
1 3 2
A= and B = ⎣2 2⎦ .
2 −1 3
3 −1
Then
12 5 12 7
AB = and (AB)T = .
7 −3 5 −3

44
1.4 Algebraic Properties of Matrix Operations 39

On the other hand,


⎡ ⎤
1 2
0 2 3
A T = ⎣ 3 −1 ⎦ and BT = .
1 2 −1
2 3
Then
12 7
B T AT = = (AB)T .
5 −3 ■

We also note two other peculiarities of matrix multiplication. If a and b are


real numbers, then ab = 0 can hold only if a or b is zero. However, this is
not true for matrices.

If
EXAMPLE 9 1 2 4 −6
A= and B= ,
2 4 −2 3
0 0
then neither A nor B is the zero matrix, but AB = . ■
0 0

If a, b, and c are real numbers for which ab = ac and a  = 0, it follows


that b = c. That is, we can cancel out the nonzero factor a. However, the
cancellation law does not hold for matrices, as the following example shows.

If
EXAMPLE 10 1 2 2 1 −2 7
A= , B= , and C = ,
2 4 3 2 5 −1
then
8 5
AB = AC = ,
16 10
but B  = C. ■

We summarize some of the differences between matrix multiplication and


the multiplication of real numbers as follows: For matrices A, B, and C of
the appropriate sizes,
1. AB need not equal B A.
2. AB may be the zero matrix with A  = O and B = O.
3. AB may equal AC with B  = C.

In this section we have developed a number of properties about matrices and


their transposes. If a future problem involves these concepts, refer to these prop-
erties to help solve the problem. These results can be used to develop many more
results.

45
40 Chapter 1 Linear Equations and Matrices

Key Terms
Properties of matrix addition Properties of scalar multiplication
Zero matrix Properties of transpose
Properties of matrix multiplication

1.4 Exercises
1. Prove Theorem 1.1(b). 11. Find two unequal 2 × 2 matrices A and B such that
2. Prove Theorem 1.1(d). 1 0
AB = .
0 1
3. Verify Theorem 1.2(a) for the following matrices:
12. Find two different 2 × 2 matrices A such that A2 = O.
1 3 −1 3 2
A= , B= , 13. Prove Theorem 1.3(a).
2 −1 1 −3 4
⎡ ⎤ 14. Prove Theorem 1.3(b).
1 0
15. Verify Theorem 1.3(b) for r = 4, s = −2, and A =
and C = ⎣ 3 −1 ⎦ .
2 −3
1 2 .
4 2
4. Prove Theorem 1.2(b) and (c). 16. Prove Theorem 1.3(c).
5. Verify Theorem 1.2(c) for the following matrices: 17. Verify Theorem 1.3(c) for r = −3,
⎡ ⎤ ⎡ ⎤
2 −3 2 0 1 2 4 2 0 2
A= , B= ,
3 −1 −2 1 3 −2 A = ⎣ 1 −3 ⎦ , and B = ⎣ 4 3⎦.
1 −3 3 2 −2 1
and C = .
−3 4 18. Prove Theorem 1.3(d).
19. Verify Theorem 1.3(d) for the following matrices:
6. Let A = ai j be the n × n matrix defined by aii = k
and ai j = 0 if i  = j. Show that if B is any n × n matrix, 1 3 −1 3 2
A= , B= ,
then AB = k B. 2 −1 1 −3 4
7. Let A be an m × n matrix and C = c1 c2 ··· cm and r = −3.
a 1 × m matrix. Prove that
20. The matrix A contains the weight (in pounds) of objects

m
CA = cj Aj, packed on board a spacecraft on earth. The objects are
j=1 to be used on the moon where things weigh about 16 as
much. Write an expression k A that calculates the weight
where A j is the jth row of A. of the objects on the moon.
cos θ sin θ 21. (a) A is a 360 × 2 matrix. The first column of A is
8. Let A = .
− sin θ cos θ cos 0 ◦ , cos 1 ◦ , . . . , cos 359 ◦ ; and the second column
is sin 0 ◦ , sin 1 ◦ , . . . , sin 359 ◦ . The graph of the or-
(a) Determine a simple expression for A2 .
dered pairs in A is a circle of radius 1 centered at
(b) Determine a simple expression for A3 . the origin. Write an expression k A for ordered pairs
(c) Conjecture the form of a simple expression for Ak , whose graph is a circle of radius 3 centered at the
k a positive integer. origin.
(d) Prove or disprove your conjecture in part (c). (b) Explain how to prove the claims about the circles in
9. Find a pair of unequal 2 × 2 matrices A and B, other than part (a).
those given in Example 9, such that AB = O. 22. Determine a scalar r such that Ax = r x, where
10. Find two different 2 × 2 matrices A such that
2 1 1
1 0 A= and x= .
A2 = . 1 2 1
0 1

46
1.4 Algebraic Properties of Matrix Operations 41

23. Determine a scalar r such that Ax = r x, where (c) For any scalar r , show that r x1 is a solution.
⎡ ⎤ ⎡ ⎤ (d) For any scalars r and s, show that r x1 + sx2 is a
1 2 −1 − 12 solution.
⎢ ⎥ ⎢ ⎥
A=⎢ ⎣1 0 1⎥ ⎢ 1⎥
⎦ and x = ⎣ 4 ⎦ . 37. Show that if Ax = b has more than one solution, then
4 −4 5 1 it has infinitely many solutions. (Hint: If x1 and x2 are
solutions, consider x3 = r x1 + sx2 , where r + s = 1.)
24. Prove that if Ax = r x for n × n matrix A, n × 1 matrix 38. Show that if x1 and x2 are solutions to the linear system
x, and scalar r , then Ay = r y, where y = sx for any Ax = b, then x1 − x2 is a solution to the associated ho-
scalar s. mogeneous system Ax = 0.
25. Determine a scalar s such that A2 x = sx when Ax = r x. 39. Let
26. Prove Theorem 1.4(a). ⎡ ⎤ ⎡ ⎤
6 −1 1 10.5
27. Prove Theorem 1.4(b) and (d). A = ⎣ 0 13 −16 ⎦ and x = ⎣ 21.0 ⎦ .
28. Verify Theorem 1.4(a), (b), and (d) for 0 8 −11 10.5
1 3 2 4 2 −1 (a) Determine a scalar r such that Ax = r x.
A= , B= ,
2 1 −3 −2 1 5 (b) Is it true that A T x = r x for the value r determined
and r = −4. in part (a)?
40. Repeat Exercise 39 with
29. Verify Theorem 1.4(c) for ⎡ ⎤
⎡ ⎤ −3.35 −3.00 3.60
3 −1
1 3 2 A = ⎣ 1.20 2.05 −6.20 ⎦
A= and B = ⎣2 4⎦.
2 1 −3 −3.60 −2.40 3.85
1 2 ⎡ ⎤
12.5
30. Let and x = ⎣ −12.5 ⎦ .
⎡ ⎤ ⎡ ⎤ ⎤⎡
2 3 −1 6.25
A = ⎣ −1 ⎦ , B = ⎣ −2 ⎦ , and C = ⎣ 5 ⎦ .
0.1 0.01
3 −4 1 41. Let A = . In your software, set the
0.001 0.0001
(a) Compute (AB T )C. display format to show as many decimal places as possi-
(b) Compute B T C and multiply the result by A on the ble, then compute
right. (Hint: B T C is 1 × 1). B = 10 ∗ A,
(c) Explain why (AB T )C = (B T C)A. C = A + A + A + A + A + A + A + A + A + A,
  
31. Determine
⎡ a constant k such that (k A)T (k A) = 1, where
⎤ 10 summands
−2 and
A = ⎣ 1 ⎦. Is there more than one value of k that could
−1 D = B − C.
be used? If D is not O, then you have verified that scalar mul-
32. Find three 2 × 2 matrices, A, B, and C such that AB = tiplication by a positive integer and successive addition
AC with B  = C and A  = O. are not the same in your computing environment. (It is
33. Let A be an n × n matrix and c a real number. Show that not unusual that D  = O, since many computing envi-
if c A = O, then c = 0 or A = O. ronments use only a “model” of exact arithmetic, called
34. Determine all 2 × 2 matrices A such that AB = B A for floating-point arithmetic.)
any 2 × 2 matrix B. 1 1
42. Let A = . In your software, set the display to
35. Show that (A − B)T = A T − B T . 1 1
show as many decimal places as possible. Experiment to
36. Let x1 and x2 be solutions to the homogeneous linear sys-
find a positive integer k such that A + 10−k ∗ A is equal
tem Ax = 0.
to A. If you find such an integer k, you have verified
(a) Show that x1 + x2 is a solution. that there is more than one matrix in your computational
(b) Show that x1 − x2 is a solution. environment that plays the role of O.

47
42 Chapter 1 Linear Equations and Matrices

1.5 Special Types of Matrices and Partitioned


Matrices
We have already introduced one special type of matrix O, the matrix all of whose
entries are zero. We now consider several other types of matrices whose structures
are rather specialized and for which it will be convenient to have special names.
An n × n matrix A = ai j is called a diagonal matrix if ai j = 0 for i  = j.
Thus, for a diagonal matrix, the terms off the main diagonal are all zero. Note
that O is a diagonal matrix. A scalar matrix is a diagonal matrix whose diagonal
elements are equal. The scalar matrix In = di j , where dii = 1 and di j = 0 for
i  = j, is called the n × n identity matrix.

Let
EXAMPLE 1
⎡ ⎤ ⎡ ⎤ ⎡ ⎤
1 0 0 2 0 0 1 0 0
A = ⎣0 2 0⎦ , B = ⎣0 2 0⎦ , and I3 = ⎣ 0 1 0 ⎦ .
0 0 3 0 0 2 0 0 1

Then A, B, and I3 are diagonal matrices; B and I3 are scalar matrices; and I3 is
the 3 × 3 identity matrix. ■

It is easy to show (Exercise 1) that if A is any m × n matrix, then

AIn = A and Im A = A.

Also, if A is a scalar matrix, then A = r In for some scalar r .


Suppose that A is a square matrix. We now define the powers of a matrix, for
p a positive integer, by
A p = A · A 
· · · · · A .
p factors

If A is n × n, we also define
A 0 = In .

For nonnegative integers p and q, the familiar laws of exponents for the real num-
bers can also be proved for matrix multiplication of a square matrix A (Exercise 8):

A p Aq = A p+q and (A p )q = A pq .

It should also be noted that the rule

(AB) p = A p B p

does not hold for square matrices unless AB = B A (Exercise 9).

48
1.5 Special Types of Matrices and Partitioned Matrices 43

An n × n matrix A = ai j is called upper triangular if ai j = 0 for i > j.


It is called lower triangular if ai j = 0 for i < j. A diagonal matrix is both upper
triangular and lower triangular.
The matrix ⎡ ⎤
EXAMPLE 2
1 3 3
A = ⎣0 3 5⎦
0 0 2
is upper triangular, and
⎡ ⎤
1 0 0
B = ⎣2 3 0⎦
3 5 2
is lower triangular. ■

A matrix A with real entries is called symmetric if A T = A.


DEFINITION 1.8

A matrix A with real entries is called skew symmetric if A T = −A.


DEFINITION 1.9
⎡ ⎤
1 2 3
A = ⎣ 2 4 5 ⎦ is a symmetric matrix. ■
EXAMPLE 3
3 5 6
⎡ ⎤
0 2 3
B = ⎣ −2 0 −4 ⎦ is a skew symmetric matrix. ■
EXAMPLE 4
−3 4 0
We can make a few observations about symmetric and skew symmetric matri-
ces; the proofs of most of these statements will be left as exercises.
It follows from the preceding definitions that if A is symmetric or skew sym-
metric, then A is a square matrix. If A is a symmetric matrix, then the entries of A
are symmetric with respect to the main diagonal of A. Also, A is symmetric if and
only if ai j = a ji , and A is skew symmetric if and only if ai j = −a ji . Moreover, if
A is skew symmetric, then the entries on the main diagonal of A are all zero. An
important property of symmetric and skew symmetric matrices is the following: If
A is an n × n matrix, then we can show that A = S + K , where S is symmetric
and K is skew symmetric. Moreover, this decomposition is unique (Exercise 29).

■ Partitioned Matrices
If we start out with an m × n matrix A = ai j and then cross out some, but not
all, of its rows or columns, we obtain a submatrix of A.
Let ⎡ ⎤
EXAMPLE 5
1 2 3 4

A = −2 4 −3 5⎦ .
3 0 5 −3

49
44 Chapter 1 Linear Equations and Matrices

If we cross out the second row and third column, we get the submatrix

1 2 4
.
3 0 −3 ■
A matrix can be partitioned into submatrices by drawing horizontal lines be-
tween rows and vertical lines between columns. Of course, the partitioning can be
carried out in many different ways.
The matrix
EXAMPLE 6
⎡ ⎤
a11 a12 a13 a14 a15
⎢a a22 a23 a24 a25 ⎥
⎢ 21 ⎥ A11 A12
A=⎢ ⎥=
⎣ a31 a32 a33 a34 a35 ⎦ A21 A22
a41 a42 a43 a44 a45

can be partitioned as indicated previously. We could also write


⎡ ⎤
a11 a12 a13 a14 a15 ⎡ ⎤
⎢a a a a a ⎥ 11 A 12 13
⎢ 21 22 23 24 25 ⎥ ⎣A A
⎦,
A=⎢ ⎥= (1)
⎣ a31 a32 a33 a34 a35 ⎦ 21 A
A 22 23
A
a41 a42 a43 a44 a45

which gives another partitioning of A. We thus speak of partitioned matrices. ■


The augmented matrix (defined in Section 1.3) of a linear system is a partitioned
EXAMPLE 7
matrix. Thus, if Ax = b, we can write the augmented matrix of this system as
A b . ■

If A and B are both m × n matrices that are partitioned in the same way, then
A + B is produced simply by adding the corresponding submatrices of A and B.
Similarly, if A is a partitioned matrix, then the scalar multiple c A is obtained by
forming the scalar multiple of each submatrix.
If A is partitioned as shown in (1) and
⎡ ⎤
b11 b12 b13 b14
⎢ ⎥ ⎡ ⎤
⎢ b21 b22 b23 b24 ⎥ B11 B12
⎢ ⎥
B=⎢ ⎥ ⎣
⎢ b31 b32 b33 b34 ⎥ = B21 B22 ,

⎢ ⎥ B31 B32
⎣ b41 b42 b43 b44 ⎦
b51 b52 b53 b54

then by straightforward computations we can show that


⎡ ⎤
11 B11 + A
(A 12 B21 + A
13 B31 ) ( A
11 B12 + A
12 B22 + A
13 B32 )
⎢ ⎥
AB = ⎣ ⎦.
22 B21 + A
21 B11 + A
(A 23 B31 ) ( A
21 B12 + A
22 B22 + A
23 B32 )

50
1.5 Special Types of Matrices and Partitioned Matrices 45

Let ⎡ ⎤
EXAMPLE 8
1 0 1 0
⎢0 2 3 −1 ⎥
⎢ ⎥ A11 A12
A=⎢ ⎥=
⎣2 0 −4 0⎦ A21 A22
0 1 0 3

and let ⎡ ⎤
2 0 0 1 1 −1
⎢ 0 1 1 −1 2 2⎥
⎢ ⎥ B11 B12
B=⎢ ⎥= .
⎣ 1 3 0 0 1 0⎦ B21 B22
−3 −1 2 1 0 −1

Then
⎡ ⎤
3 3 0 1 2 −1
⎢ 6 12 0 −3 7 5⎥
⎢ ⎥ C11 C12
AB = C = ⎢ ⎥= ,
⎣ 0 −12 0 2 −2 −2 ⎦ C21 C22
−9 −2 7 2 2 −1

where C11 should be A11 B11 + A12 B21 . We verify that C11 is this expression as
follows:

1 0 2 0 0 1 0 1 3 0
A11 B11 + A12 B21 = +
0 2 0 1 1 3 −1 −3 −1 2

2 0 0 1 3 0
= +
0 2 2 6 10 −2

3 3 0
= = C11 .
6 12 0 ■
This method of multiplying partitioned matrices is also known as block mul-
tiplication. Partitioned matrices can be used to great advantage when matrices
exceed the memory capacity of a computer. Thus, in multiplying two partitioned
matrices, one can keep the matrices on disk and bring into memory only the sub-
matrices required to form the submatrix products. The products, of course, can be
downloaded as they are formed. The partitioning must be done in such a way that
the products of corresponding submatrices are defined.
Partitioning of a matrix implies a subdivision of the information into blocks, or
units. The reverse process is to consider individual matrices as blocks and adjoin
them to form a partitioned matrix. The only requirement is that after the blocks
have been joined, all rows have the same number of entries and all columns have
the same number of entries.
Let
EXAMPLE 9
2 9 8 −4
B= , C = 1 −1 0 , and D= .
3 6 7 5

51
46 Chapter 1 Linear Equations and Matrices

Then we have
⎡ ⎤
9 8 −4
2 9 8 −4 D
B D = , = ⎣6 7 5⎦ ,
3 6 7 5 C
1 −1 0

and
⎡ ⎤
9 8 −4 1
D
CT = ⎣6 7 5 −1 ⎦ .
C
1 −1 0 0 ■

Adjoining matrix blocks to expand information structures is done regularly in


a variety of applications. It is common for a business to keep monthly sales data
for a year in a 1 × 12 matrix and then adjoin such matrices to build a sales history
matrix for a period of years. Similarly, results of new laboratory experiments are
adjoined to existing data to update a database in a research facility.
We have already noted in Example 7 that the augmented matrix of the linear
system Ax = b is a partitioned matrix. At times we shall need to solve several
linear systems in which the coefficient matrix A is the same, but the right sides of
the systems are different, say, b, c, and d. In these cases we shall find it convenient
to consider the partitioned matrix A b c d . (See Section 4.8.)

■ Nonsingular Matrices
We now come to a special type of square matrix and formulate the notion corre-
sponding to the reciprocal of a nonzero real number.
An n × n matrix A is called nonsingular, or invertible, if there exists an n × n
DEFINITION 1.10
matrix B such that AB = B A = In ; such a B is called an inverse of A. Otherwise,
A is called singular, or noninvertible.

Remark In Theorem 2.11, Section 2.3, we show that if AB = In , then B A = In .


Thus, to verify that B is an inverse of A, we need verify only that AB = In .
   
3
2 3 −1 2 . Since AB = B A = I , we conclude that
Let A = and B = 2
EXAMPLE 10 2 2 1 −1
B is an inverse of A. ■

Theorem 1.5 The inverse of a matrix, if it exists, is unique.


Proof
Let B and C be inverses of A. Then
AB = B A = In and AC = C A = In .
We then have B = B In = B(AC) = (B A)C = In C = C, which proves that the
inverse of a matrix, if it exists, is unique. ■

52
1.5 Special Types of Matrices and Partitioned Matrices 47

Because of this uniqueness, we write the inverse of a nonsingular matrix A as


A−1 . Thus
A A−1 = A−1 A = In .
Let
EXAMPLE 11 1 2
A= .
3 4
If A−1 exists, let
a b
A−1 = .
c d
Then we must have
1 2 a b 1 0
A A−1 = = I2 = ,
3 4 c d 0 1
so that
a + 2c b + 2d 1 0
= .
3a + 4c 3b + 4d 0 1
Equating corresponding entries of these two matrices, we obtain the linear systems
a + 2c = 1 b + 2d = 0
and
3a + 4c = 0 3b + 4d = 1.

The solutions are (verify) a = −2, c = 32 , b = 1, and d = − 12 . Moreover, since


the matrix    
a b −2 1
= 3
c d 2
− 12
also satisfies the property that
    
−2 1 1 2 1 0
3 = ,
2
− 12 3 4 0 1
we conclude that A is nonsingular and that
 
−2 1
A−1 = 3 .
− 12
2 ■
Let
EXAMPLE 12 1 2
A= .
2 4
If A−1 exists, let
a b
A−1 = .
c d
Then we must have
1 2 a b 1 0
A A−1 = = I2 = ,
2 4 c d 0 1

53
48 Chapter 1 Linear Equations and Matrices

so that
a + 2c b + 2d 1 0
= .
2a + 4c 2b + 4d 0 1
Equating corresponding entries of these two matrices, we obtain the linear systems

a + 2c = 1 b + 2d = 0
and
2a + 4c = 0 2b + 4d = 1.

These linear systems have no solutions, so our assumption that A−1 exists is in-
correct. Thus A is singular. ■

We next establish several properties of inverses of matrices.

Theorem 1.6 If A and B are both nonsingular n × n matrices, then AB is nonsingular and
(AB)−1 = B −1 A−1 .
Proof
We have (AB)(B −1 A−1 ) = A(B B −1 )A−1 = (AIn )A−1 = A A−1 = In . Similarly,
(B −1 A−1 )(AB) = In . Therefore AB is nonsingular. Since the inverse of a matrix
is unique, we conclude that (AB)−1 = B −1 A−1 . ■

Corollary 1.1 If A1 , A2 , . . . , Ar are n × n nonsingular matrices, then A1 A2 · · · Ar is nonsingular


and (A1 A2 · · · Ar )−1 = Ar−1 Ar−1 −1
−1 · · · A1 .

Proof
Exercise 44. ■

Theorem 1.7 If A is a nonsingular matrix, then A−1 is nonsingular and (A−1 )−1 = A.
Proof
Exercise 45. ■

Theorem 1.8 If A is a nonsingular matrix, then A T is nonsingular and (A−1 )T = (A T )−1 .


Proof
We have A A−1 = In . Taking transposes of both sides, we get

(A−1 )T A T = InT = In .

Taking transposes of both sides of the equation A−1 A = In , we find, similarly, that

(A T )(A−1 )T = In .

These equations imply that (A−1 )T = (A T )−1 . ■

If
EXAMPLE 13
1 2
A= ,
3 4

54
1.5 Special Types of Matrices and Partitioned Matrices 49

then from Example 11


  ⎡ ⎤
3
−2 1 −2
A−1 = 3 and (A−1 )T = ⎣ 2 ⎦.
2
− 12 1 − 12

Also (verify),
⎡ ⎤
3
1 3 −2
AT = and (A T )−1 = ⎣ 2 ⎦.
2 4 1 − 12

Suppose that A is nonsingular. Then AB = AC implies that B = C (Exer-
cise 50), and AB = O implies that B = O (Exercise 51).
It follows from Theorem 1.8 that if A is a symmetric nonsingular matrix, then
A−1 is symmetric. (See Exercise 54.)

■ Linear Systems and Inverses


If A is an n × n matrix, then the linear system Ax = b is a system of n equations in
n unknowns. Suppose that A is nonsingular. Then A−1 exists, and we can multiply
Ax = b by A−1 on the left on both sides, yielding
A−1 (Ax) = A−1 b
(A−1 A)x = A−1 b
In x = A−1 b
x = A−1 b. (2)
Moreover, x = A−1 b is clearly a solution to the given linear system. Thus, if A is
nonsingular, we have a unique solution. We restate this result for emphasis:

If A is an n × n matrix, then the linear system Ax = b has the unique solution


x = A−1 b. Moreover, if b = 0, then the unique solution to the homogeneous
system Ax = 0 is x = 0.

If A is a nonsingular n × n matrix, Equation (2) implies that if the linear


system Ax = b needs to be solved repeatedly for different b’s, we need compute
A−1 only once; then whenever we change b, we find the corresponding solution
x by forming A−1 b. Although this is certainly a valid approach, its value is of a
more theoretical rather than practical nature, since a more efficient procedure for
solving such problems is presented in Section 2.5.
Suppose that A is the matrix of Example 11 so that
EXAMPLE 14  
−1 −2 1
A = 3 .
2
− 12
If
8
b= ,
6

55
50 Chapter 1 Linear Equations and Matrices

then the solution to the linear system Ax = b is


    
−2 1 8 −10
x = A−1 b = 3 = .
2
− 12 6 9

On the other hand, if


10
b= ,
20
then
10 0
x = A−1 = .
20 5 ■

■ Application A: Recursion Relation; the Fibonacci Sequence


In 1202, Leonardo of Pisa, also called Fibonacci,∗ wrote a book on mathematics
in which he posed the following problem: A pair of newborn rabbits begins to
breed at the age of 1 month, and thereafter produces one pair of offspring per
month. Suppose that we start with a pair of newly born rabbits and that none of
the rabbits produced from this pair dies. How many pairs of rabbits will there be
at the beginning of each month?
At the beginning of month 0, we have the newly born pair of rabbits P1 . At the
beginning of month 1 we still have only the original pair of rabbits P1 , which have
not yet produced any offspring. At the beginning of month 2 we have the original
pair P1 and its first pair of offspring, P2 . At the beginning of month 3 we have the
original pair P1 , its first pair of offspring P2 born at the beginning of month 2, and
its second pair of offspring, P3 . At the beginning of month 4 we have P1 , P2 , and
P3 ; P4 , the offspring of P1 ; and P5 , the offspring of P2 . Let u n denote the number
of pairs of rabbits at the beginning of month n. We see that
u 0 = 1, u 1 = 1, u 2 = 2, u 3 = 3, u 4 = 5, u 5 = 8.
The sequence expands rapidly, and we get
1, 1, 2, 3, 5, 8, 13, 21, 34, 55, 89, 144, . . .
To obtain a formula for u n , we proceed as follows. The number of pairs of
rabbits that are alive at the beginning of month n is u n−1 , the number of pairs
who were alive the previous month, plus the number of pairs newly born at the
beginning of month n. The latter number is u n−2 , since a pair of rabbits produces
a pair of offspring, starting with its second month of life. Thus
u n = u n−1 + u n−2 . (3)

∗ Leonardo Fibonacci of Pisa (about 1170–1250) was born and lived most of his life in Pisa, Italy.
When he was about 20, his father was appointed director of Pisan commercial interests in northern
Africa, now a part of Algeria. Leonardo accompanied his father to Africa and for several years traveled
extensively throughout the Mediterranean area on behalf of his father. During these travels he learned
the Hindu–Arabic method of numeration and calculation and decided to promote its use in Italy. This
was one purpose of his most famous book, Liber Abaci, which appeared in 1202 and contained the
rabbit problem stated here.

56
1.5 Special Types of Matrices and Partitioned Matrices 51

That is, each number is the sum of its two predecessors. The resulting sequence of
numbers, called a Fibonacci sequence, occurs in a remarkable variety of applica-
tions, such as the distribution of leaves on certain trees, the arrangements of seeds
on sunflowers, search techniques in numerical analysis, the generation of random
numbers in statistics, and others.
To compute u n by the recursion relation (or difference equation) (3), we have
to compute u 0 , u 1 , . . . , u n−2 , u n−1 . This can be rather tedious for large n. We now
develop a formula that will enable us to calculate u n directly.
In addition to Equation (3), we write

u n−1 = u n−1 ,

so we now have

u n = u n−1 + u n−2
u n−1 = u n−1 ,

which can be written in matrix form as


un 1 1 u n−1
= . (4)
u n−1 1 0 u n−2

We now define, in general,

u k+1 1 1
wk = and A= (0 ≤ k ≤ n − 1)
uk 1 0

so that
u1 1
w0 = = ,
u0 1
u2 2 u n−1 un
w1 = = ,..., wn−2 = , and wn−1 = .
u1 1 u n−2 u n−1

Then (4) can be written as


wn−1 = Awn−2 .
Thus
w1 = Aw0
w2 = Aw1 = A(Aw0 ) = A2 w0
w3 = Aw2 = A(A2 w0 ) = A3 w0
..
.
wn−1 = An−1 w0 .

Hence, to find u n , we merely have to calculate An−1 , which is still rather tedious if
n is large. In Chapter 7 we develop a more efficient way to compute the Fibonacci
numbers that involves powers of a diagonal matrix. (See the discussion exercises
in Chapter 7.)

57
52 Chapter 1 Linear Equations and Matrices

Key Terms
Diagonal matrix Symmetric matrix Nonsingular (invertible) matrix
Identity matrix Skew symmetric matrix Inverse
Powers of a matrix Submatrix Singular (noninvertible) matrix
Upper triangular matrix Partitioning Properties of nonsingular matrices
Lower triangular matrix Partitioned matrix Linear system with nonsingular coefficient matrix
Fibonacci sequence

1.5 Exercises
1. (a) Show that if A is any m × n matrix, then Im A = A 12. For a nonsingular matrix A and a nonnegative integer p,
and AIn = A. show that (A p )−1 = (A−1 ) p .
(b) Show that if A is an n × n scalar matrix, then 13. For a nonsingular matrix A and nonzero scalar k, show
A = r In for some real number r . that (k A)−1 = k1 A−1 .
2. Prove that the sum, product, and scalar multiple of diag- 14. (a) Show that every scalar matrix is symmetric.
onal, scalar, and upper (lower) triangular matrices is di- (b) Is every scalar matrix nonsingular? Explain.
agonal, scalar, and upper (lower) triangular, respectively. (c) Is every diagonal matrix a scalar matrix? Explain.
3. Prove: If A and B are n × n diagonal matrices, then 15. Find a 2 × 2 matrix B = O and B = I2 such that
AB = B A. 1 2
AB = B A, where A = . How many such matri-
4. Let 2 1
⎡ ⎤ ⎡ ⎤ ces B are there?
3 2 −1 6 −3 2
16. Find a 2 × 2 matrix B  = O and B  = I2 such that
A = ⎣0 −4 3⎦ and B = ⎣0 2 4⎦.
1 2
0 0 0 0 0 3 AB = B A, where A = . How many such matri-
0 1
Verify that A + B and AB are upper triangular. ces B are there?
5. Describe all matrices that are both upper and lower trian- 17. Prove or disprove: For any n ×n matrix A, A T A = A A T .
gular. 18. (a) Show that A is symmetric if and only if ai j = a ji
for all i, j.
1 2 1 −1
6. Let A = and B = . Compute each (b) Show that A is skew symmetric if and only if ai j =
3 −2 2 3
of the following: −a ji for all i, j.
(a) A2 (b) B 3 (c) (AB)2 (c) Show that if A is skew symmetric, then the elements
⎡ ⎤ ⎡ ⎤ on the main diagonal of A are all zero.
1 0 −1 0 0 1
7. Let A = ⎣ 2 1 1 ⎦ and B = ⎣ −1 1 1 ⎦. 19. Show that if A is a symmetric matrix, then A T is sym-
metric.
3 1 0 2 0 1
Compute each of the following: 20. Describe all skew symmetric scalar matrices.
(a) A3 (b) B 2 (c) (AB)3 21. Show that if A is any m × n matrix, then A A T and A T A
are symmetric.
8. Let p and q be nonnegative integers and let A be a square
matrix. Show that 22. Show that if A is any n × n matrix, then
(a) A + A T is symmetric.
A p Aq = A p+q and (A p )q = A pq .
(b) A − A T is skew symmetric.
9. If AB = B A and p is a nonnegative integer, show that 23. Show that if A is a symmetric matrix, then Ak , k =
(AB) p = A p B p . 2, 3, . . . , is symmetric.
10. If p is a nonnegative integer and c is a scalar, show that 24. Let A and B be symmetric matrices.
(c A) p = c p A p . (a) Show that A + B is symmetric.
11. For a square matrix A and a nonnegative integer p, show (b) Show that AB is symmetric if and only if AB =
that (A T ) p = (A p )T . B A.

58
1.5 Special Types of Matrices and Partitioned Matrices 53

25. (a) Show that if A is an upper triangular matrix, then Find the solution x.
A T is lower triangular. 38. The linear system A2 x = b is such that A is nonsingular
T
(b) Show that if A is a lower triangular matrix, then A with
is upper triangular. 3 0 −1
A−1 = and b = .
26. If A is a skew symmetric matrix, what type of matrix is 2 1 2
A T ? Justify your answer. Find the solution x.
27. Show that if A is skew symmetric, then the elements on 39. The linear system A T x = b is such that A is nonsingular
the main diagonal of A are all zero. with
28. Show that if A is skew symmetric, then Ak is skew sym- 4 1 1
A−1 = and b = .
metric for any positive odd integer k. 1 0 −2
29. Show that if A is an n ×n matrix, then A = S + K , where Find the solution x.
S is symmetric and K is skew symmetric. Also show that
40. The linear system C T Ax = b is such that A and C are
this decomposition is unique. (Hint: Use Exercise 22.)
nonsingular, with
30. Let ⎡ ⎤
1 3 −2 2 1 0 2 1
A = ⎣4 6 2⎦. A−1 = , C −1 = , and b= .
3 1 1 4 1
5 1 3
Find the matrices S and K described in Exercise 29. Find the solution x.
2 3 41. Consider the linear system Ax = b, where A is the ma-
31. Show that the matrix A = is singular. trix defined in Exercise 33(a).
4 6
⎡ ⎤ 3
4 0 0 (a) Find a solution if b = .
4
32. If D = ⎣ 0 −2 0 ⎦, find D −1 .
0 0 3 5
(b) Find a solution if b = .
33. Find the inverse of each of the following matrices: 6
1 3 1 2 42. Find two 2 × 2 singular matrices whose sum is nonsin-
(a) A = (b) A = gular.
5 2 2 1
43. Find two 2 × 2 nonsingular matrices whose sum is sin-
2 1
34. If A is a nonsingular matrix whose inverse is , gular.
4 1
find A. 44. Prove Corollary 1.1.
35. If 45. Prove Theorem 1.7.
46. Prove that if one row (column) of the n ×n matrix A con-
3 2 2 5
A−1 = and B −1 = , sists entirely of zeros, then A is singular. (Hint: Assume
1 3 3 −2 that A is nonsingular; that is, there exists an n × n matrix
B such that AB = B A = In . Establish a contradiction.)
find (AB)−1 .
47. Prove: If A is a diagonal matrix with nonzero di-
36. Suppose that
agonal entries a11 , a22 , . . . , ann , then A is nonsingu-
1 2
A−1 = . lar and A−1 is a diagonal matrix with diagonal entries
1 3 1/a11 , 1/a22 , . . . , 1/ann .
Solve the linear system Ax = b for each of the following ⎡ ⎤
2 0 0
matrices b:
48. Let A = ⎣ 0 −3 0 ⎦. Compute A4 .
4 8
(a) (b) 0 0 5
6 15
49. For an n × n diagonal matrix A whose diagonal entries
37. The linear system ACx = b is such that A and C are
are a11 , a22 , . . . , ann , compute A p for a nonnegative inte-
nonsingular with
ger p.
2 1 2 1 2 50. Show that if AB = AC and A is nonsingular, then
A−1 = , C −1 = , and b = .
B = C.
−1 1 1 2 3

59

You might also like